Jump to content

Wikipedia:Reference desk/Miscellaneous

From Wikipedia, the free encyclopedia

This is an old revision of this page, as edited by Polysylabic Pseudonym (talk | contribs) at 04:40, 13 April 2010 (→‎Hemp powder). The present address (URL) is a permanent link to this revision, which may differ significantly from the current revision.

Welcome to the miscellaneous section
of the Wikipedia reference desk.
Select a section:
Want a faster answer?

Main page: Help searching Wikipedia

   

How can I get my question answered?

  • Select the section of the desk that best fits the general topic of your question (see the navigation column to the right).
  • Post your question to only one section, providing a short header that gives the topic of your question.
  • Type '~~~~' (that is, four tilde characters) at the end – this signs and dates your contribution so we know who wrote what and when.
  • Don't post personal contact information – it will be removed. Any answers will be provided here.
  • Please be as specific as possible, and include all relevant context – the usefulness of answers may depend on the context.
  • Note:
    • We don't answer (and may remove) questions that require medical diagnosis or legal advice.
    • We don't answer requests for opinions, predictions or debate.
    • We don't do your homework for you, though we'll help you past the stuck point.
    • We don't conduct original research or provide a free source of ideas, but we'll help you find information you need.



How do I answer a question?

Main page: Wikipedia:Reference desk/Guidelines

  • The best answers address the question directly, and back up facts with wikilinks and links to sources. Do not edit others' comments and do not give any medical or legal advice.
See also:


April 7

Déjà vu by multiple people at the same time?

So A, B and I were in a car. As we were driving over a bridge B threw an apple core out the window into the river. I said I felt like this had happened before. We all experience Déjà vu sometimes so I'm used to it. However A and B both said that they felt like it had happened before too, that they were experiencing Déjà vu also. That's the first and only (and last) time we've everthrown anything in to that river, especially from a moving vehicle. Wtf happened?--92.251.159.250 (talk) 00:25, 7 April 2010 (UTC)[reply]

Some possibilities:
1) A coincidence. That is, you both had a similar "tossing an object" experience.
2) Person B is putting person A on. That is, it's not really a déjà vu for them.
3) The power of suggestion. Person B again didn't really have a déjà vu, but person A saying so convinced them that they did, too. This is how mass hysteria starts. StuRat (talk) 00:54, 7 April 2010 (UTC)[reply]
4) You really have thrown something out the window before that's triggering a common feeling. I suppose it need not have been in that specific river or from that specific vehicle. Buddy431 (talk) 00:56, 7 April 2010 (UTC)[reply]
As the article on Déjà vu says, the cause may not be a confusion with an actual memory from the past, but a glitch in our perception of the present which accidentally processes the short-term memory (of the last second or two) as if it came from long-term memory. (This doesn't address why you all had it at once, I'm just saying. And of course confusion with a real memory might add to the feeling.) 81.131.64.245 (talk) 01:15, 7 April 2010 (UTC)[reply]
Someone is changing the Matrix! Run! Adam Bishop (talk) 04:14, 7 April 2010 (UTC)[reply]
Perhaps you were stuck in a repeating "time loop" like on a Star Trek Next Generation episode, where eventually each participant in a card game "knew" what cards would come up next. Edison (talk) 14:35, 7 April 2010 (UTC)[reply]

Lemon martini (talk) 00:10, 8 April 2010 (UTC) ;)[reply]

You're sure you'd already posted something, right?! --Polysylabic Pseudonym (talk) 05:12, 8 April 2010 (UTC)[reply]
whoa! deja vu!!! http://comics.com/monty/2010-04-07/ !!!Gzuckier (talk) 06:31, 8 April 2010 (UTC)[reply]

How can this car move?

http://punditkitchen.com/2009/10/21/political-pictures-attack-fail/

It seems that this car is warproof. Anyways...

  1. Can you identify the car's model? I'm just curious.
  2. How can this thing be moving at all? Does it run on willpower or something?

--121.54.2.188 (talk) 08:09, 7 April 2010 (UTC)[reply]

Don't know which model it is, but the image caption claims it is a Toyota. However, the badge on the car's front grille, doesn't look like the badge used on Toyotas for the last many years. As for how it can be moving at all, how do you know it was driven at all. Maybe the vehicle was towed, the photo staged, or the soldiers just came across a couple of locals sitting in a car wreck. Even so, cars can still be driven with a substantial amount of bodywork damage so long as the mechanical parts still work and the wheels turn without shredding the tyres. The car in the image looks to have been involved in a crash or maybe rolled, rather then the damage being the result of a hit by some kind of weapon. Whatever happened to it, it was an incident in which the driver was probably lucky to survive judging by the extent of the damage. Astronaut (talk) 09:28, 7 April 2010 (UTC)[reply]
Agreed. Looks like mostly body damage to me. Granted, it won't pass a US state inspection but I don't see an obvious reason why it wouldn't move under its own power. Dismas|(talk) 09:32, 7 April 2010 (UTC)[reply]
It looks to me like it was side-swiped by something big and high, like an MRAP, that tore off the side panels and roof. However, the components which make a car drivable are under the hood, on the dashboard, under the car, and the wheels, and they all look to be intact. StuRat (talk) 09:47, 7 April 2010 (UTC)[reply]
The main problem would be the structural integrity of the thing. Most (intentional) convertibles have extra bracing in the chassis and more welds in the bodywork than hard-tops. Without that, I'd be worried about it failinng catastrophically at speed. However, without a windshield, I doubt they'll be driving very fast! But I don't see any reason why the engine & drivetrain shouldn't work perfectly. Of course there is always the possibility that this is just a clever fake. SteveBaker (talk) 03:10, 8 April 2010 (UTC)[reply]
I cannot tell the exact model. But it's a Corolla van. Probably Toyota Corolla (E100), Toyota Corolla (E110) or Toyota Corolla (E120). Oda Mari (talk) 06:22, 8 April 2010 (UTC)[reply]
i've seen videos of FWD cars which were cut in half behind the front seats and still drove around fine with their hinders scooting on the ground like an impacted dog. of course, they had to be slightly modified re fuel tanks, etc but the car in the pic looks to be in pretty good shape, third world wise. Gzuckier (talk) 06:34, 8 April 2010 (UTC)[reply]

looking for advice

So, I'm writing a book, as you do, and one of my characters is having some trouble concentrating on their work, motivating themselves to get homework done and so on. A friend of theirs offers a lot of advice that helps, but i have little idea what that advice should be, so far I have only the suggestion to eat breakfast every day and get regular exercise. I would like for the advice they are given to be real, enough that people reading the book could follow it themselves.

Later they arrange a system where, on the first day they go for a short walk, then gradually longer walks, before starting jogging, gradually building up toward more difficult exersizes later. But it seems, not being a particularly physically active person myself, I have little idea what they could be doing after that. Preferably it should be things to help them become more healthy, rather than practicing and gaining skill in a particular sport. So, any ideas?

80.47.88.128 (talk) 10:18, 7 April 2010 (UTC)[reply]

For the exercise régime, perhaps they could come across a copy of the RCAF 5BX exercise programme. Our article has a link to the programme, with descriptions of the exercises involved. DuncanHill (talk) 10:28, 7 April 2010 (UTC)[reply]
As for motivation, how about a series of small rewards ? Like, when they get their homework done, they get to eat a cookie ? StuRat (talk) 11:31, 7 April 2010 (UTC)[reply]
I know this will not look like the most helpful response, but for the exercise bit, why not give it a shot yourself before writing it? The best authors write what they know, and learn what they don't know but want to write about. Rather than writing something about how easy it is to scale up an exercise regimen from nothing, why not give it a shot yourself first? It'll give you deeper insights into how it works and how your characters would view the situation. It'll produce more realistic writing and more realistic characters, especially if you are hoping your readers will be able to actually get something out of reading it. --Mr.98 (talk) 14:05, 7 April 2010 (UTC)[reply]

Port on BMW bumper

What is this little port on the rear bumper of some BMWs? If it's for servicing the car in some way, why is it in such a prominent spot? Thanks. --Sean 15:05, 7 April 2010 (UTC)[reply]

WAG: It may have something to do with the advanced vehicle detection systems in newer models of luxury cars, for example some cars are equipped with "back up cameras" that show you the view behind your car; others have proximity sensors to tell you when you are going to hit something behind you, and they either sound a warning or automatically apply the brakes for you. My guess is that this has something to do with one of those sorts of systems. --Jayron32 15:21, 7 April 2010 (UTC)[reply]
If it's like my MINI (BMW), opening the cover reveals a threaded hole for screwing in a tow hook. There's a similar port in the front grill. -- Coneslayer (talk) 15:23, 7 April 2010 (UTC)[reply]
It is indeed a rear tow point. The tow hook screws in once you remove the little panel. See this link. Dismas|(talk) 19:32, 7 April 2010 (UTC)[reply]
WARNING: It's not intended for towing the car serious distances - it's just enough to winch the car up onto the bed of a tow truck or to pull you out when you get stuck in snow or mud or something. You should find the eyelet/hook thingy somewhere in with the tools for changing the wheel, etc. It's exceedingly dangerous to use it for long distance towing because the hook can easily become unthreaded! (One of the members of my local car club found this out to his cost when towing his car just over a mile to the nearest garage!) SteveBaker (talk) 20:40, 7 April 2010 (UTC)[reply]
yeah, it's not literally a tow hook, it's a tie-down hook to keep the car on top of the carrier which is actually toting it around. Gzuckier (talk) 06:35, 8 April 2010 (UTC)[reply]
On page 113 of the owner's manual for my '09 MINI Cooper'S (which is a BMW design) calls it a "tow fitting" and talks about towing the car using it. When I've had cars put on the back of the carrier, they've chained up the axles and put those nylon webbing things around the wheels...I've only seen them use the tow fitting to pull the car onto the bed of the truck. But other car manufacturers may be different. SteveBaker (talk) 03:06, 9 April 2010 (UTC)[reply]
Thanks to all for the (surprising) answer! --Sean 15:12, 8 April 2010 (UTC)[reply]

hepatitis a

My partner developed hepatits a and jaundice in pakistan and was hospitalised. Is there a posibility that i could contract hepatitis a from him? —Preceding unsigned comment added by Kathyhollyoake (talkcontribs) 15:26, 7 April 2010 (UTC)[reply]

The answer to any "is it possible?" type of question is typically going to be "almost anything is possible". To get a useful answer, please see a doctor. You could also read the Hepatitis article. ←Baseball Bugs What's up, Doc? carrots15:29, 7 April 2010 (UTC)[reply]
Or even Hepatitis A. 130.126.109.178 (talk) 17:26, 7 April 2010 (UTC)[reply]

Made in Korea

If I buy a product in the US and it has the label "Made in Korea", what are the chances that it was made in North Korea as opposed to South Korea? Googlemeister (talk) 18:13, 7 April 2010 (UTC)[reply]

Pretty much zero. StuRat (talk) 18:15, 7 April 2010 (UTC)[reply]
When it comes to sports federations, for example, "Korea" usually refers to the South (Korea Republic). North Korea is usually called "DPR Korea" or "Korea DPR" or something to that effect. Rimush (talk) 18:44, 7 April 2010 (UTC)[reply]
Yes, with the D being for Democratic. I wonder, does any country which is actually democratic put that in it's name ? Or is it the same effect as "Honest Al's Discount TVs" (if Al were really honest, he wouldn't need to convince people by putting it in his name). StuRat (talk) 19:24, 7 April 2010 (UTC)[reply]
You made me wonder that too, so I tried to find out. Defining democracy in terms of Polity IV, the only democracy with "Democratic" in its full name that I can find is Sri Lanka. --Tango (talk) 20:06, 7 April 2010 (UTC)[reply]
And how many non-democratic Democratic nations did you find ? StuRat (talk) 01:39, 8 April 2010 (UTC)[reply]
A dozen or so. --Tango (talk) 02:18, 8 April 2010 (UTC)[reply]
Well, that certainly is a lopsided enough ratio to support the "Honest Al" theory. StuRat (talk) 03:31, 8 April 2010 (UTC)[reply]
Yes, it is. (A dozen or so is all of them - I went through List of countries searching for "demo".) --Tango (talk) 16:48, 8 April 2010 (UTC)[reply]
What about the Democratic Republic of the Congo? 92.30.145.113 (talk) 23:21, 7 April 2010 (UTC)[reply]
According to our Democratic Republic of the Congo article, "Mobutu institutionalized corruption to prevent political rivals from challenging his control". They now have new leadership and a new constitution as of 2006, and corruption is down somewhat, so maybe they can now be considered truly democratic. StuRat (talk) 01:35, 8 April 2010 (UTC)[reply]
I think one should reserve judgement for more than 4 years before declaring a country democratic. They've only had the one general election so far, that's not really enough to see if they are going to make democracy work. --Tango (talk) 02:18, 8 April 2010 (UTC)[reply]
A country where the military rapes and kills at will is not democratic. Rimush (talk) 09:32, 8 April 2010 (UTC)[reply]
What if everyone votes that the military should be allowed to rape and kill at will? Or, at least, what if the raping and killing does not interfere with the process of free elections? "Democratic" does not necessarily mean "good and just", just that people get to vote. APL (talk) 16:25, 8 April 2010 (UTC)[reply]
Heroin? SteveBaker (talk) 20:32, 7 April 2010 (UTC)[reply]
In Prague, I once saw a Cold War-era coffee cup marked "MADE IN D.P.R.K." In English! -- Mwalcoff (talk) 00:03, 8 April 2010 (UTC)[reply]
100% for nuclear bombs and missiles. Just don't let Jack Bauer hear about it. Clarityfiend (talk) 03:22, 8 April 2010 (UTC)[reply]
I once came across a website where you could find DPRK stuff, primarily propaganda like cartoons and books IIRC and stuff to indicate your support for the DPRK. I don't know how much, if any of it was actually made in the DPRK of course Nil Einne (talk) 02:47, 12 April 2010 (UTC)[reply]

I was just looking for pretty pictures and came across this one, look at the page and under the heading rational there is a picture od Buzz, and it states that he is a chriastina, what relavance does this have to tghe rest of the article, I would try to find out for my self, and am , there are 2 ways to go about doing this, 1 is to read it my self, and 2 is to read your responses tomorrow —Preceding unsigned comment added by 62.172.58.82 (talk) 19:05, 7 April 2010 (UTC)[reply]

The caption says "Astronaut Buzz Aldrin, a Christian, had a personal Communion service when he first arrived on the surface of the Moon". It's describing the extension of religion into space, which is arguably part of space exploration. I suppose one could argue that it belongs in a separate article, though (along with our recent question about which direction Muslim astronauts pray). StuRat (talk) 19:21, 7 April 2010 (UTC)[reply]
Since those are the only two interesting facts we have about religion in space, I'm not sure it really warrants an article. It might warrant its own section in the space exploration article, I suppose. --Tango (talk) 19:30, 7 April 2010 (UTC)[reply]
<shrug> we already have an article about Sex in space - so why not?! SteveBaker (talk) 20:27, 7 April 2010 (UTC)[reply]
Maybe we need an article about Golf on the moon, starting (and ending, so far) with Alan Shepard's attempt to turn the moon into a driving range. ←Baseball Bugs What's up, Doc? carrots23:03, 7 April 2010 (UTC)[reply]
I don't know about two facts. An 18 page guidebook was published for Muslims in space as part of the Angkasawan programme so there's probably a decent amount you could write about that. [1] I know things which came up include fasting and prayer times and the ref also mentions ablution. I believe we've discussed some of this a while back (I'm not referring to the mention of one aspect referred to above). And there are other things like kosher food and celebrating the Jewish shabbat [2]. Edit: Wait why are we even talking about this? We already have a Religion in space and it's existed long before this discussion. P.S. Golf in space Nil Einne (talk) 02:33, 12 April 2010 (UTC)[reply]
A third interesting fact is mentioned at Apollo 8#Historical importance — the astronauts read from Genesis and NASA was subsequently sued, and although the lawsuit was dismissed, it apparently made NASA skittish about public displays of religion in space by these government employees, so Aldrin's Communion was private. (Side note: I think this belongs very, very low on the list of reasons Apollo 8 was "historically important".) Comet Tuttle (talk) 23:20, 7 April 2010 (UTC)[reply]
There is no law against government employees displaying religious beliefs (or lack thereof). But maybe they should have had a disclaimer like "these views do not necessarily reflect those of the management." ←Baseball Bugs What's up, Doc? carrots23:54, 7 April 2010 (UTC)[reply]
True, but they aren't allowed to make it appear to a reasonable observer that the government is officially endorsing (or favoring) any religion, and the 3 most prominent Americans in the world at that moment reciting Genesis on their way back from the first human orbit around another world, with the US having spent billions of dollars to elevate them to the position where they do that — this reasonable observer has a problem with that. Comet Tuttle (talk) 01:29, 8 April 2010 (UTC)[reply]
As an American watching on TV at the time, it never occurred to me that their reading from Genesis was anything more than their personal expression of how they felt at the time. ←Baseball Bugs What's up, Doc? carrots18:57, 9 April 2010 (UTC)[reply]
Considering all the taxpayer money it would cost to lift even a crucifix into orbit, the atheists may have a point. StuRat (talk) 01:48, 8 April 2010 (UTC)[reply]
In case anyone misses it above, we already have religion in space Nil Einne (talk) 02:48, 12 April 2010 (UTC)[reply]

EDLP

Hi. I have just been reading an article about pricing structure in supermarkets. It turns out that "EDLP" is one strategy that supermarkets use to lure so-called "large basket shoppers". The quote is:

Furthermore, EDLP stores get a greater than expected share of business from large basket    
shoppers; HILO stores get a greater than expected share from small basket shoppers. 

Does anyone know what EDLP stands for ? Robinh (talk) 20:24, 7 April 2010 (UTC)[reply]

Every Day Low Pricing. SteveBaker (talk) 20:28, 7 April 2010 (UTC)[reply]
(ec) This link says it's "Every-Day-Low-Price" and the other principal strategy is apparently HILO, the "high-low strategy". Everyday low price is our stubby article on EDLP. Comet Tuttle (talk) 20:29, 7 April 2010 (UTC)[reply]
Thanks guys. Obvious when you know. Robinh (talk) 20:44, 7 April 2010 (UTC)[reply]
I just rewrote the Everyday low price article, mostly based on 2 one-page web pages. If you have further information (on EDLP, or on the Hi-Lo strategy, which has no article yet), please go for it. Comet Tuttle (talk) 20:56, 7 April 2010 (UTC)[reply]
I guess the quote makes sense, because people who shop for sales will only get the few items they want which are on sale at the HILO, then go to the everyday low price stores for everything else. I tend to do this myself. StuRat (talk) 01:22, 8 April 2010 (UTC)[reply]
Is HILO advertising the loss-leaders but having higher prices on everything else? 78.146.107.183 (talk) 20:11, 12 April 2010 (UTC)[reply]


April 8

French people

Why aren't French people fat? Their food has lots of fat, oil, sugar, etc. --70.129.184.122 (talk) 01:07, 8 April 2010 (UTC)[reply]

See French Paradox#French diet comparisons. Clarityfiend (talk) 01:14, 8 April 2010 (UTC)[reply]
All that smoking and adultery ? :-) StuRat (talk) 01:17, 8 April 2010 (UTC)[reply]
Ah, le Tigre Woods diet. Clarityfiend (talk) 01:44, 8 April 2010 (UTC)[reply]
Some of them are fat...(oh crap! my (french) wife is coming - quick, pretend someone else typed this!)
NO! You are completely wrong! All french people are definitely very, very thin! (Sheesh - don't you hate it when people don't sign their posts?) SteveBaker (talk) 02:56, 8 April 2010 (UTC)[reply]
You may want to read the book French Women Don't Get Fat by Mireille Guiliano. I haven't read the book, but I think it's worth pointing out that Europeans don't drive everywhere like Americans so often do, living in cities with little parking. -- Mwalcoff (talk) 03:14, 8 April 2010 (UTC)[reply]
in my experiences of France (mostly Normandy region) people don't seem to care about 'little parking' - they just park anywhere there is a space..or to put it more accurately they'll just nudge your car forward (with theirs) until it creates a space big enough for their car! 194.221.133.226 (talk) 08:27, 8 April 2010 (UTC)[reply]
One place I saw (slightly) fatter French people was at huge out-of-town shopping malls where you have to drive to get there, there's ample parking and plenty of fast food. Umm, the dangers of an american lifestyle. Astronaut (talk) 03:42, 8 April 2010 (UTC)[reply]
Who's thin? [3] Richard Avery (talk) 06:02, 8 April 2010 (UTC)[reply]
Must be in their genes.--Rallette (talk) 11:01, 8 April 2010 (UTC)[reply]
At least some of them will be eating a Mediterranean diet, which is apparently very good for you. Alansplodge (talk) 16:00, 9 April 2010 (UTC)[reply]
I don't know who left that unsigned post, but need I remind people that SB has an obvious WP:COI and so shouldn't be trusted in this instance? Nil Einne (talk) 01:48, 10 April 2010 (UTC)[reply]

The Right Honourable Seán Dublin Bay Rockall Loftus

How did Seán Dublin Bay Rockall Loftus manage to become Lord Mayor without being affiliated to one of the major political parties? 86.45.133.15 (talk) 02:16, 8 April 2010 (UTC)[reply]

Lord Mayor of Dublin is an symbolic position, it doesn't really mean much, so there is no particular reason for the council to choose one of its members from a major political party. It is also not unusual for independents to do well in local elections (party policies are usually to do with national issues, so aren't relevant), so him being on the council isn't at all strange. --Tango (talk) 02:30, 8 April 2010 (UTC)[reply]
And, in general, there are all sorts of reasons why a party might nominate an independent, or someone from another party, for such a post. If they don't think that one of their own members can win the post, then they might support someone with similar views to them, or tactically choose someone unaffiliated or from a minor party to prevent a major opposing party from taking the post. Alternatively, they might nominate someone in return for political concessions - support on one or more issues. For a symbolic post, they might choose someone in order to reduce their time for meaningful political activity; for a difficult post, they might see it as a "poisoned chalice" and choose someone in order to threaten their popularity. Warofdreams talk 10:07, 8 April 2010 (UTC)[reply]

True north

Is there any kind of print or image that can be used to find true north?.. I've heard of using a watch and what not but I was wondering if there is something that could be tattooed on my skin so that I could always find true north . Thanks for any help 188.81.80.78 (talk) 14:05, 8 April 2010 (UTC)[reply]

I think not. IIRC, the watch trick depends upon knowing what time it is & where the sun is ... the two together allow you to figure out true north based on the sun's position. A static image just does not have the necessary information. --Tagishsimon (talk) 14:13, 8 April 2010 (UTC)[reply]
For the watch trick, see direction finding watch. --Richardrj talk email 14:24, 8 April 2010 (UTC)[reply]
You could always tattoo a star chart on your skin (several, if you want to be good for any location/any season), but in the northern hemisphere it's pretty easy to find the pole star anyway, which is as close to north as you'll get by eye. In the southern hemisphere, I guess you could find the South pole, and take the opposite direction as north. There's no pole star though, but I think you can get pretty close using Crux somehow. Buddy431 (talk) 14:36, 8 April 2010 (UTC)[reply]
A simple map would also allow you to find true north, provided that you can locate and identify the landmarks from the map. StuRat (talk) 17:49, 8 April 2010 (UTC)[reply]
Use a compass with a magnetic declination adjustment, set properly for your geographic region. 66.127.52.47 (talk) 18:00, 8 April 2010 (UTC)[reply]
If the sun or moon is clear enough to cast shadows, put a stick in the ground so that its shadow vanishes, and wait a few minutes; when the shadow moves, it will point due east. (I learned this from a television character who was making a point about the value of reading.) —Tamfang (talk) 19:15, 8 April 2010 (UTC) — Come to think of it, he didn't say due. —Tamfang (talk) 15:57, 10 April 2010 (UTC)[reply]
Pity the TV character didn't know what they were talking about, then. This is based on the idea that the Sun's motion in the sky is westward. That's only true near solar noon. 6 hours later, if it's still up, it's moving north or south, depending on which hemisphere you're in. --Anonymous, 19:28 UTC, April 8, 2010.
Hm. The daily motion is always in a plane perpendicular to the axis ... I'll have to try it. :( —Tamfang (talk) 04:20, 9 April 2010 (UTC)[reply]
Meanwhile, as an existence proof, consider a location in the Arctic summer with 24-hour daylight. Near solar midnight the sun is in the north and is moving eastward, as it circles around the sky day after day. In the mid-latitudes we just lose the northern part of that circle as it goes below ground. --Anon, 06:04 UTC, April 9, 2010.

A Wikipedia article explains how to use a banana as a compass. Place a banana on the Berlin Wall. East is where a bite has been taken out of it. Cuddlyable3 (talk) 23:50, 8 April 2010 (UTC)[reply]

Assuming you're in the Northern Hemisphere as your IP address suggests, try looking for Polaris in the night sky. If you cross the equator, Southern Cross explains how to find South. Zoonoses (talk) 00:17, 9 April 2010 (UTC)[reply]
Telling South by the sun without knowing the time (if you've got all day to spare): put a stick in the ground so that it's standing upright, then mark where the tip of the shadow falls at regular(ish) intervals. The direction where the shadow was shortest is South (or North if you're in the southern hemisphere). Alansplodge (talk) 15:22, 9 April 2010 (UTC)[reply]
Vice versa. The shadow points away from the sun, eh? --Anonymous, 23:30 UTC, April 9, 2010.
Generally vice-versa, yes, although the direction will be reversed in summer at latitudes lower than the subsolar point. FiggyBee (talk) 06:54, 13 April 2010 (UTC)[reply]
IIRC, moss growing on tree trunks can also be used as a rough indication of which way is north. Moss prefers a shady spot and therefore is more prevalent on the side of the trunk facing away from the midday sun. Astronaut (talk) 17:43, 9 April 2010 (UTC)[reply]
Perhaps very generally, but it is probably not a good idea to rely on that indication heavily, as the moss tends to grow in the more shadowy side, which, depending on local geography, might not be the north side. Googlemeister (talk) 18:51, 9 April 2010 (UTC)[reply]
If you looked at where the moss was on a few hundred trees spread over a reasonably wide area, that might work, but it's more of an urban legend than a real wilderness technique. --Tango (talk) 16:02, 10 April 2010 (UTC)[reply]
I was taught the moss one in Cubs, and also the urban equivalent. In this part of the world, satellite TV dishes point south :-) 93.97.184.230 (talk) 22:13, 11 April 2010 (UTC)[reply]

symbols

There is a symbol on my shower gel bottle and shampoo and on numerous others it is a picture od a round, flat jar with the lid floating off it, and on the side is written 12m, does any one know what this is? what does it mean? it is in black and white next to the recycle symbol. Any Ideas? —Preceding unsigned comment added by 62.172.59.90 (talk) 15:30, 8 April 2010 (UTC)[reply]

I guess it is inducating the product should last for 12 months after being opened. I think I'm right in thinking that such markings are increasingly common on non-food items. I'll try to find a reference. --Tagishsimon (talk) 15:35, 8 April 2010 (UTC)[reply]
Period-after-opening symbol. --Tagishsimon (talk) 15:36, 8 April 2010 (UTC)[reply]

period after opening symbol, thats it thanks —Preceding unsigned comment added by 62.172.59.90 (talk) 18:35, 8 April 2010 (UTC)[reply]

InterRail question

I plan on making an InterRail journey this summer to be finally able to visit the World Bodypainting Festival, and to stop at a tourist destination along the way, without once travelling by airplane. I have already planned a journey from Stockholm to Kraków, from there to Spittal an der Drau, and from there back to Stockholm. The problem is, the trip from Stockholm to Kraków takes about a full day, the trip from Kraków to Spittal an der Drau takes a bit more than half a day, and the trip from Spittal an der Drau to Stockholm takes a bit less than a day and a half. That's more travelling days than the minimal InterRail pass allows for, and I feel that the next expensive pass is overkill, because I don't intend on spending that many days. I have thought that one option would be to instead use the trip from Vienna to Spittal an der Drau as a normal, non-InterRail railway trip. Surely this is possible? Do I have any other options? Please note that I am intentionally trying to avoid ever boarding an airplane during the entire trip. JIP | Talk 19:28, 8 April 2010 (UTC)[reply]

Certainly there's nothing forcing you to use your InterRail card if you don't want to - an ordinary adult 2nd class ticket from Vienna Westbahnhof to Spittal-Millstaettersee will cost around €59 one way. (Krakow to Spittal via the Zebrzydow border route/Breclav/Vienna costs €102.20 2nd class - see http://www.jizdenka.cz/). The only thing you have to ensure is that you have enough valid days on your InterRail to cover the other journeys that you want to make. -- Arwel Parry (talk) 21:20, 8 April 2010 (UTC)[reply]
Two things. As Arwel pointed out, you don't need to use the Interrail pass for all journeys and are free to purchase separate tickets for sections. If you do travel by train internationally, it's much cheaper to purchase a ticket just to the border, and then to get off at the border and purchase the ticket for the section in the other country (this is usually possible, the trains usually wait a while, i. e. between Poland and the Czech Republic I've done this myself; make sure i. e. at the international ticket office at the main railway station in Kraków that this is the case for your connection; when in doubt find someone Polish-speaking to assist you, the English of the people working there can sometimes be terrible). Also, if you need to, have you considered hitch-hiking? It can be difficult in Austria, I know, but Poland and the Czech Republic are quite hitch-hiker-friendly places. --Ouro (blah blah) 11:53, 9 April 2010 (UTC)[reply]
It has been many years since I had an Interrail pass and hitch-hiked around Europe (two different trips), but if I were you, I might choose a segment outside of Austria, preferably in Poland or the Czech Republic, to travel on a regular ticket without the Interrail pass. The reason is that Austria has (or used to have) much higher rail fares than Poland or the Czech Republic. As for hitch-hiking, as a young male, I found it could be great fun. (It is riskier for women.) It is helpful to 1) stand in a place where a driver can see you from some distance and where it is easy and legal for a driver to stop, 2) to have a sign indicating your destination or general direction in the local language (e.g. "Vídeň" for Vienna if you are in the Czech Republic), and 3) to be clean-cut looking and fairly conservatively dressed (try to look like a nice student who is going to visit his grandparents but is too poor for the train). Marco polo (talk) 17:23, 9 April 2010 (UTC)[reply]

Thanks for the replies. I figure that what I might do is catch a late evening train (by InterRail) from Kraków to Vienna, and then a morning train (as a normal trip) from Vienna to Spittal an der Drau. The InterRail rules state that if an overnight train leaves after 19:00, it only takes up one travelling day, provided it arrives after 04:00 the next day. This, however, applies to that specific train only, connecting trips do take up an extra travelling day. 59 € doesn't sound so bad for the trip from Vienna to Spittal an der Drau. It's only a bit less than double the cost of a train trip from Helsinki to Tampere, with the distance being roughly one and a half times as long. And considering I would be travelling in a foreign country, I would want to minimise the hassle and risks involved. That would allow me to use the return trip from Spittal an der Drau to Stockholm completely by InterRail, which would save me a lot of money. JIP | Talk 19:47, 9 April 2010 (UTC)[reply]

Umm, I'm not quite sure if the effect is what you want. If I read you correctly, you want to travel from Kraków starting late evening on day "A", arriving Vienna in the morning of day "B", then travel on to Spittal later on day "B". I don't know if InterRail works the same way FIP (rail staff) tickets do (which I use), but in that circumstance I would date the ticket with day "B"s date, since you're starting the journey after 19:00, so the whole journey to Spittal would be covered by InterRail. The question that then becomes relevant is 'are you planning to do any travel earlier on day "A", or will you have stayed in Krakow for a day or more?'. I'd suggest you have another read of the InterRail rules to check which date you should be putting on the ticket. Regards, -- Arwel Parry (talk) 00:01, 11 April 2010 (UTC)[reply]
Looks like I had misread that part of the InterRail rules and thought I had to mark day "A" as the travelling day and arriving in Vienna on day "B" on the same train would be covered, but not the trip from Vienna to Spittal an der Drau. I reread the rules and found out that I'm allowed to mark day "B" instead. This solves my original problem - I simply have to leave Kraków after 19:00 on day "A", and I can use as much of day "B" to get to Spittal an der Drau as I want. JIP | Talk 03:26, 11 April 2010 (UTC)[reply]

Webshop user behaviour logging (Not Google Analytics or similar)

I have no experience with e-commerce on the sellers side, but I was wondering how much 'logging' control an administrator of a webshop site has when using a commercial webshop platform. Ie. by admin I mean the guys selling stuff using a webshop.

And does some of the webshop platforms allow 'live' logging of user behaviour?

Thanks 85.81.121.107 (talk) 19:50, 8 April 2010 (UTC)DB[reply]

Yes, you could have a lot of logging, you could have it live. Even basic weblogging (just looking at requests from users) gets you a lot of information, much of it real-time. If you had a more sophisticated (AJAX) solution, you could find out exactly what they were clicking on the moment they clicked. There is some information you won't get, because the browser won't give it to you, but Javascript will give a LOT of information away, and that could all be harvested for someone watching on the server end of things. Note that this information would generally only be available to whomever is hosting the website—it's not like I, person with no connection to you or the sites you visit, could get this information. But if you visit another website, they can get a lot. They already do, quite regularly. --Mr.98 (talk) 22:27, 8 April 2010 (UTC)[reply]
Though I should add, there are limits. They can't, for example, read stuff off your hard drive. The browsers won't give them that information. But the limits are a lot less than most people know—people are often surprised that browsers can give real-time browser information, OS information, screen resolution, things of that nature. Even mouse position, window position, mouse movement, etc. But only within the site's own window—they can't (generally speaking) see what you are doing with other sites. (Cookies being the one main exception to this in some cases.) --Mr.98 (talk) 22:29, 8 April 2010 (UTC)[reply]

Sugar, sugar!

Is there a difference betweem confectioners sugar and powdered sugar? —Preceding unsigned comment added by 99.137.227.240 (talk) 19:54, 8 April 2010 (UTC)[reply]

No, see confectioner's sugar. StuRat (talk) 21:13, 8 April 2010 (UTC)[reply]
Some people make a distinction between the two based on the presence/absence of anticaking agent (commonly cornstarch). For most practical purposes, though, they can be used interchangeably. -- 140.142.20.229 (talk) 17:09, 9 April 2010 (UTC)[reply]


April 9

Do you eat at a table and not do other things while eating?

Just wondering how many people sit at a table while eating and don't watch tv or use a computer or w/e while eating.--92.251.166.223 (talk) 00:39, 9 April 2010 (UTC)[reply]

Well, if there are other people at the table, I talk with them. If not, then I wouldn't eat at the table at all, but in front of the TV and/or computer. StuRat (talk) 00:43, 9 April 2010 (UTC)[reply]
It's just my wife and I, so we eat on the couch while watching TV and the dogs watch us while waiting for handouts. Dismas|(talk) 01:02, 9 April 2010 (UTC)[reply]
I only sit at the table if we have guests. We should really try to find some survey results rather than having our own survey, though, since this is a ref desk! --Tango (talk) 01:20, 9 April 2010 (UTC)[reply]
You can see some survey figures here. -- Wavelength (talk) 03:13, 9 April 2010 (UTC)[reply]
When I am alone (and when I was single) I eat (and ate) in front of the computer always practically. When me and my girlfriend eat together, we eat in the kitchen, where there's no TV or anything. We avoid eating and watching TV or anything at the same time, except for snacks during films. --Ouro (blah blah) 11:47, 9 April 2010 (UTC)[reply]
A bunch of Wikipedia editors probably isn't a very representative sample of anything (except Wikipedia editors, I suppose). Still, I almost always eat at a table. When I'm alone, I read a newspaper. Buddy431 (talk) 14:47, 9 April 2010 (UTC)[reply]
Wow buddy same here, I was a bit surprised when I heard someone saying most people don't, although I hhavent' seen any actual statistics before now. Thanks.--92.251.159.197 (talk) 16:29, 9 April 2010 (UTC)[reply]
If with company, with relatives or visiting people, I definitely prefer to sit at a table and talk (and so it seems do the other people I eat with). However, when alone at home I usually eat in front of the TV (though in my old house, which was much larger than the current place I live, I often ate at the table with the newspaper in front of me). Astronaut (talk) 17:29, 9 April 2010 (UTC)[reply]

public access tunnel directly under capitol building for vehicles

My son claims a few years ago he drove directly under the capitol building thru a public access tunnel.

I disagree it was directly under the building. Perhaps nearby under capitol grounds but not directly under building. Who's right.

Thankyou.

(Mrs.) Nancy A. Baughman <contact info redacted> —Preceding unsigned comment added by 70.60.182.65 (talk) 02:22, 9 April 2010 (UTC)[reply]

Yeah - go to Google maps and type "Capitol building, Washington DC" - scroll over to the left and you'll see that highway 395 goes under the "reflecting pool" to the west of the Capitol building. It doesn't go right under the Capitol building though - so I guess you're right. SteveBaker (talk) 02:42, 9 April 2010 (UTC)[reply]
There are, however, tunnels that do go directly under the Capitol building, just not accessible by the general public, except by invitation. See United States Capitol subway system. --Jayron32 03:48, 9 April 2010 (UTC)[reply]
(OR) Many years ago (pre-9/11), I was a reporter doing a story in the Capitol. I got lost and found myself down in those tunnels, and it took forever to figure out how to get back. — Michael J 22:08, 9 April 2010 (UTC)[reply]

Perhaps you need to be a Stonecutter to access them Lemon martini (talk) 19:38, 10 April 2010 (UTC)[reply]

woman

When I am trying to initiate intercourse with my girlfriend, I either cuddle her and kiss her and then try to take her shirt off. Or I start to touch her, well recently, she will stop me and tell me to go to sleep. So I do. Then she will wake me up and yell at me that we naver have sex. I tell her that last night I tried to take her shirt off and she asked me what I was doing so I said I am trying to take your clothes off so we can make love, but you told me to stop and go to sleep so I did, now you are angry because I stopped when you told me to. So my question is, when a woman tells you no stop you have to stop right? If not it is rape, right? I don't understand what I have done wrong, it would appear that she wants me to force myself on her, but I am not going to do that. I am a gentle person and would like to make sweet love, what am I missing here? Does she want me to force her? Maybe a woman can help. This relationship is over, as I ran away this morning and am not going back, but my prev girlfriend only ever wanted sex in the doggy style, I don't understand this either. Please help so I can improve future relationships. Thanks —Preceding unsigned comment added by 93.186.23.206 (talk) 10:55, 9 April 2010 (UTC)[reply]

"'No' always means no . . ..'No' has always been, and always will be, part of the dangerous, alluring courtship ritual of sex and seduction, observable even in the animal kingdom" - Camille Paglia, Source. Though telling you to go to sleep is a pretty forceful "no". —Preceding unsigned comment added by Droptone (talkcontribs) 13:27, 9 April 2010 (UTC)[reply]
I would suggest looking somewhere else. You don't need to put up with that crap. There are 3 billion females on this planet. Googlemeister (talk) 14:10, 9 April 2010 (UTC)[reply]
Agreed. She's jerking him around, and he was wise to flee. I'm guessing that when she complained, instead of arguing with her about the previous night, if he had said, "OK, let's do it right now," she would have found a reason not to. However, one alternative to fleeing could have been to say, "I don't understand", and put the ball back in her court. ←Baseball Bugs What's up, Doc? carrots14:30, 9 April 2010 (UTC)[reply]
What Baseball Bugs says about doing it right now is probably very true. She might have some problems of some kind that you should have helped, but there's no way of knowing. But then there is noone who doesn't have problems of some kind.--92.251.159.197 (talk) 16:33, 9 April 2010 (UTC)[reply]
She probably needed more romance, like telling her she's the prettiest girl on Earth, who has ever lived, or will ever live, and also in all parallel universes. If you can get through that without cracking up, she may open up. StuRat (talk) 14:41, 9 April 2010 (UTC)[reply]
The most romantic, sexiest thing you can do for a woman is clean the house, cook the meal, sit her down with a drink and tell her she means the world to you. --TammyMoet (talk) 15:00, 9 April 2010 (UTC)[reply]
I want a source for this. --Ouro (blah blah) 15:33, 9 April 2010 (UTC)[reply]
No it isn't, it is obviously a great thing to do but classic romantic stuff is better.--92.251.159.197 (talk) 16:30, 9 April 2010 (UTC)[reply]
You wanna source? Try any married woman, or woman who has to look after a home as well as work, kids, big kids (i.e. men)... We don't fall for classic romantic stuff. We know you're only after one thing if you go down on one knee - usually expecting us to do the same! --TammyMoet (talk) 18:59, 9 April 2010 (UTC)[reply]
You've got it right. The "traditional" romantic stuff needs to be done also, but it's not enough. Exceeding expectations once in awhile will work wonders. Am I right? :) ←Baseball Bugs What's up, Doc? carrots19:34, 9 April 2010 (UTC)[reply]
Well that depends on your home situation, if like most relationships the woman does the housework well then probably. However I clean up after myself and she cleans up after herself and we split hte rest of the jobs such as mowing the lawn (me) and emptying the bin (her).--92.251.159.197 (talk) 19:48, 9 April 2010 (UTC)[reply]
And if you did her chores sometimes, especially if she's weary (as with the song "Try a Little Tenderness"), you could score big-time points by doing "her" work for her, as it would exceed expectations. If you have a bright line dividing "your" work and "her" work that's never to be crossed, that's more of a "roommate" relationship than a "love" relationship. ←Baseball Bugs What's up, Doc? carrots20:01, 9 April 2010 (UTC)[reply]
Welcome to the world of women! Women are not consistent creatures, you just have to accept that. You can try talking to them and trying to understand, but your chances of success are never high. --Tango (talk) 15:15, 9 April 2010 (UTC)[reply]
And lest anyone get too concerned, one can say the same thing about men. Human beings are not consistent creatures. Relationships are often fraught. A good relationship is usually the one in which you and your partner are crazy in the same kind of way. --Mr.98 (talk) 15:54, 9 April 2010 (UTC)[reply]
Or perhaps in mutually complimentary ways, such as a sadist and a masochist. StuRat (talk) 16:23, 9 April 2010 (UTC)[reply]

The one time she came to my house I cleaned it so it was spotless, so I could show her that my house is not as much of a mess as hers, she was 3 hours late and then said that she had already eaten, so I ate alone by candle light, she then also said that she does not want to come to my house as it is to far to travel, I should know I spend every second day at her house and have to do the commute! Later in the relationship I told her I had to work at 6am the next day, so I gave her a 2 hour massage and then stated that I need to go to sleep because I am working, she then started shouting at me that we never spend anytime together, and continued on until 3am. I was then very tired but agreed to go back to her house that night after work, I fell asleep as you do when you have only had 2 hours sleep the night before, she then wakes me up at 4am as she wants to have a cigarette and does not want to smoke alone. And now this stuff above, man I think I have been used! Reading it now, it seems so strange that I put up with all this if only for 1.5 months. She acts like she is a super model, and she certainly is not. The small amout of sex was dreadful, and I am a man so it must have been bad. Well thanks wikipedians, onwards and upwards. —Preceding unsigned comment added by 62.172.58.82 (talk) 16:29, 9 April 2010 (UTC)[reply]

To my mind, she's playing a game called, "You can't win." So, as with the movie War Games, "the only way to win is not to play." I'm also reminded at this point of a Fifth Dimension song called "Carpet Man". You did the right thing by leaving. But I'll also say this - cooking and cleaning for a woman are very romantic things to do - for the right woman, i.e. one who appreciates it. ←Baseball Bugs What's up, Doc? carrots17:06, 9 April 2010 (UTC)[reply]
Reminds me of Should I Stay or Should I Go, by The Clash: [4]. StuRat (talk) 17:51, 9 April 2010 (UTC)[reply]
A very wise man once said (paraphrasing) "If you are in a relationship with a woman, you can be right or you can be happy. But not both at the same time." --Jayron32 18:42, 9 April 2010 (UTC)[reply]
Womenfolk and menfolk have different conceptions of what the relationship is. Bus stop (talk) 19:25, 9 April 2010 (UTC)[reply]
Bus Stop is hereby given the Understatement of the Day Award. :) ←Baseball Bugs What's up, Doc? carrots19:32, 9 April 2010 (UTC)[reply]
Indeed. And on the goals of an arguement within the relationship. A woman's goal is to win the arguement. A man's goal is to not be in the arguement. Those are easily resolved goals as long as the man is willing to always lose. --Jayron32 19:41, 9 April 2010 (UTC)[reply]


From My Little Chickadee:

W.C. Fields: "I never argue with a lady!"
Mae West: "Smart boy!"

Baseball Bugs What's up, Doc? carrots19:36, 9 April 2010 (UTC)[reply]

Funny how most (possibly all except Tammy Moet) of the people here giving us the good oil about the arcane ways of women, are men. Men have always misunderstood women, and vice-versa. That's at least part of the point of being different. (I gave up even trying years ago.) So, I'm not entirely sure their comments, well intentioned as they are, can be completely trusted. But on the other hand, asking a woman to reveal her secrets, or expecting there to be quality information available in a reputable source ... sorry, it doesn't work that way. There's a reason why there has to be this tension between men and women. Harry knew what he was talking about. Inherent uncertainty and unpredictability and confusion are part of the game. It's a woman's biological role to frustrate men - apparently it keeps their sexual energy high or something. Add to that the individual characteristic of being an outright bitch, and you've got quite a problem on your hands. (Not saying your girlfriend is one of those, but some women do take their role as Arch-Frustratrix uber-seriously.) Unfortunately, it's not the sort of problem that anonymous people on the internet, particularly males, can help you with.  :) -- Jack of Oz ... speak! ... 21:23, 9 April 2010 (UTC)[reply]

I know. Start calling her Trixie. She'll think it's cute and affectionate. Only you will know what it really stands for. Unless she also reads this page.  :) -- Jack of Oz ... speak! ... 21:26, 9 April 2010 (UTC)[reply]
Oh, yeh, that could work. Hey, it's already almost a lost cause anyway. Jerry Seinfeld once said a couple of cogent things on this subject (many cogent things, actually). One is that men and women are like firemen and fire, respectively. Men can be ready at any time, like an emergency. Woman can get really hot, but the conditions have to be right. The other thing he said was that in terms of attracting women, "Honking horns and yelling out the window are [unfortunately] the best ideas we've come up with so far." ←Baseball Bugs What's up, Doc? carrots22:07, 9 April 2010 (UTC)[reply]

Headphone clip is broken

The little clip on my headphones which is used to attach the cord to your shirt or jacket so that it doesn't get in your way is broken (see pic, it's blurred but you get the idea). I have still got the other part of the clip but I have failed to glue it back on using superglue. The clip is hardwired to the headphones so it's not possible to buy a replacement part, and I'm certainly not going to buy a whole new pair of headphones just for that. Please provide suggestions as to how I can attach the black plastic bit you see in the picture to my clothes without the clip, thank you. --Richardrj talk email 14:51, 9 April 2010 (UTC)[reply]

Carefully cut apart a pen lid (of the appropriate shape - some rely on the pen body as a 'back', other clips are all on the lid) and manipulate/glue as necessary? Or attach a paperclip/safety pin as necessary? 94.168.184.16 (talk) 14:55, 9 April 2010 (UTC)[reply]
If using a paper clip (or butterfly clip), be sure to use a plastic one, as the sharp ends on metal ones might snag on clothing. StuRat (talk) 16:20, 9 April 2010 (UTC)[reply]
I would probably glue something like an alligator clip to it with superglue. --Mr.98 (talk) 17:00, 9 April 2010 (UTC)[reply]
(ec)A small alligator clip might work. In fact, there's a plastic one on my own cellphone's headset. Less likely to break, I suspect, than the item the OP refers to. ←Baseball Bugs What's up, Doc? carrots17:01, 9 April 2010 (UTC)[reply]

SIPHON PUMPS

A [SIPHON PUMP] is defined as being a device consisting of a conduit bent and forming legs of unequal length, this conduit has an inline holding canister at the upper bend. The action of pressure of the atmosphere forces liquid up the shorter leg of the conduit immersed in it, while the continued excess weight of the liquid in the longer branch causes a continuous flow. Withdrawal of the liquid at the upper bend is metered when both the outlet and inlet of the siphon are shut off, the canister is emptied of it's contents and shut air tight. The siphon flow begins when the flow valves open and allow for the purging of trapped air below the source liquid inlet. The SIPHON PUMP will withdraw liquid repeatedly from the canister that can be positioned above the source as disclosed in US Patent # 5358000. —Preceding unsigned comment added by 12.155.114.134 (talk) 16:09, 9 April 2010 (UTC)[reply]

Yes, and why have you posted this here ? Do you have a question for us ? StuRat (talk) 16:15, 9 April 2010 (UTC)[reply]
Perhaps the OP thinks we should have an article on siphon pump, but a siphon and a pump are different things, so that expression is misleading; we have a comprehensive article about the siphon, which is what he/she means.--Shantavira|feed me 17:18, 9 April 2010 (UTC)[reply]

Homosexual male referring to himself as "Miss"?

I just heard Gok Wan reffered to on TV as "Miss Gok Wan". He's male. I would very much like to be reffered to as "Dr." or "Sir", or even better, "Your Highness". I wouldn't get away with that, so why is he allowed to call himself this?--92.251.159.197 (talk) 16:41, 9 April 2010 (UTC)[reply]

Freedom of speech. ←Baseball Bugs What's up, Doc? carrots16:58, 9 April 2010 (UTC)[reply]
"Dr.", "Sir", and "Your Highness" are all titles that only people who have attained a certain status can use. For example, a person needs to earn a doctoral degree at an institution of higher learning to legally call himself or herself "Dr." The titles "Sir" and "Your Highness" can only be used by people who are monarchs or who have been recognized for their achievements by a monarch. In contrast, there are no legal restrictions on who may call himself or herself "Miss". You can call yourself "Miss" too if you like. Marco polo (talk) 17:04, 9 April 2010 (UTC)[reply]
If you are going to take seriously anything Gok Wan says then I think you deserve whatever annoyance you receive. Come on, he's almost ficticious! Caesar's Daddy (talk) 07:52, 10 April 2010 (UTC)[reply]
I'd like to see your source for there being a legal restriction to calling yourself "Dr." or "Sir" or any other title in the US. If you were to impersonate a doctor or defraud someone, then I could see the issue but simply referring to yourself as a doctor, I've never known that to be illegal. Dr. John seems to get away with it. As well as Dr. Johnny Fever. Dismas|(talk) 17:16, 9 April 2010 (UTC)[reply]
Yes, you can call yourself pretty much anything you want to in the USA, as long as it's not for fraudulent reasons or violates some specific law. I don't think "Doc" Gooden has an advanced degree. (Although he seems to dabble in pharmacology from time to time.)Baseball Bugs What's up, Doc? carrots17:21, 9 April 2010 (UTC)[reply]
Neither do Dr. Demento, Dr. Dre, or "The Doctor" of Doctor and the Medics. – ClockworkSoul 18:58, 10 April 2010 (UTC)[reply]
It's not at all unusual or a recent phenomenon for gay men to refer to themselves facetiously as "girls" or whatever. In the 1960s movie The Boys in the Band, for example. ←Baseball Bugs What's up, Doc? carrots17:12, 9 April 2010 (UTC)[reply]
Yeah but "Mister", "Misses" and "Miss" convey certain information about the person, so they're being difficult just for the sake of it. If they want to be women let them get transexual procedures done.--92.251.159.197 (talk) 17:33, 9 April 2010 (UTC)[reply]
See Self-determination. Ultimately, its up to the person what they want to call themselves. It's not really either your business or your problem. When a person asks to be called "Miss" or "Mister", neither really causes measurable harm to you. So I'm not sure you have a grievance here... --Jayron32 18:01, 9 April 2010 (UTC)[reply]
Yes, self-determination. And I call what the OP is saying "nannyism". The late, great Alan King said, "The world is full of little dictators trying to run your life." Those little dictators, as he called them, are often called "nannies". You're being rather blunt by telling the OP it's none of his business, but guess what - you're right. That Gok Wan guy isn't being "difficult", he's just doing what he feels like doing in order to be funny and entertaining. And whether he ever chooses to get a sex-change operation is entirely his own concern. I don't think there are all that many gay men who want to give up their "equipment" - even the gay men who call themselves "girls". ←Baseball Bugs What's up, Doc? carrots18:13, 9 April 2010 (UTC)[reply]
I would add gender identity as a more specific article on the topic. -- Coneslayer (talk) 18:15, 9 April 2010 (UTC)[reply]
The essence of the OP's complaint is "Why doesn't Gok Wan change something for my convenience?" And that kind of viewpoint is the essence of nannyism. Now, if Gok Wan's TV ratings were to drop off, specifically due to him calling himself that, then he might change it. The best way to get someone to change something is to appeal to their selfish interests. Telling them to change something just because you don't like it is not likely to get any response beyond an MYOB. I don't see anything suggesting that Gok Wan would gain anything by dropping that nickname and/or lose anything by retaining it. ←Baseball Bugs What's up, Doc? carrots18:21, 9 April 2010 (UTC)[reply]
I think most people are rather missing the point. Wasn't this on Sharon Osbourne's A Comedy Roast last night? In which case, it was the job of everybody there to mock everyone else - especially Sharon, but certainly other comedians are fair game. I could be wrong...but I assumed that was it. Vimescarrot (talk) 18:35, 9 April 2010 (UTC)[reply]

I find it a little annoying that he would want this, but only because he's telling me what to call him. My definition of 'Miss' does not include men, so if he wants me to call him that, he can change his name. He has the freedom to want to be called whatever he wants: We have the freedom not to call him that.Aaronite (talk) 18:39, 9 April 2010 (UTC)[reply]

And who's forcing you to call him that, or trying to? ←Baseball Bugs What's up, Doc? carrots18:45, 9 April 2010 (UTC)[reply]
You are. You're massively overreacting, no need to get so defensive, I'm simply asking if this is a common phenonemom, and stating that I don't particularly like it because it's confusing. Similarly I wouldn't like someone calling a pot a pan. I am not suggesting there be legislation or any other crap, which would be absolutely ridiculous, and which is what you are defending against. Surely you would have a problem if I set up a really smelly and noisy meat factory just opposite your home? Does that make you a nanny? No. I also have problems with uptight people such as yourself jumping the gun just because of someone's opinion. Do I find his title odd, confusing and slightly irritating? Definitely. Have I suggested legislation or other action to "force" Gok Wan to call himself Mister? No I haven't. Would I? No, never. If anyone is being a nanny it's you, in your attempts to so vigorously destroy our opinions.--92.251.159.197 (talk) 19:46, 9 April 2010 (UTC)[reply]
I'm... what? Forcing you to call him that? No, I'm not forcing you to do anything. You can call him whatever you want to. Now, if he's a personal friend, you should maybe call him what he asks you to call him, as a matter of courtesy. But since you find "Miss" confusing, I have to assume that the two of you are not buds. In fact, why is it confusing at all? He's just being funny. ←Baseball Bugs What's up, Doc? carrots19:58, 9 April 2010 (UTC)[reply]
During that show there were jokes about him but I assume that wasn't one as nobody laughed.--92.251.159.197 (talk) 20:20, 9 April 2010 (UTC)[reply]
Probably not, if it's what he normally calls himself. But you know he's a man and that he likes to call himself "Miss" for whatever reason. So where's the confusion? ←Baseball Bugs What's up, Doc? carrots20:24, 9 April 2010 (UTC)[reply]
Welcome to the modern age. The issue here is that "Miss Gok Wan" has (apparently) a female sexual identity, and would like to be referenced that way. you are under no obligation to do so, of course, but that's what it is. This is different from being gay - gay men often have a masculine sexual identity with a sexual preference for other males, rather than a sexual identity of being female. title like 'Doctor', 'Sir', or 'Your Highness' are conferred according to particular academic or social qualifications - no one is under any obligation to use those, either, mind you - but sexual identity labels are supposed to accord to the person's inherent sexual identity. You simply disagree with "Miss Gok Wan" about the nature of his/her inherent sexual identity, and the question is: what grounds do you have to disagree with him/her on what sexual identity he/she has? --Ludwigs2 20:44, 9 April 2010 (UTC)[reply]
Near as I can tell, the "confusion" boils down to the fact that Gok Wan won't conform to convention. That's not "confusion", it's "conformism". ←Baseball Bugs What's up, Doc? carrots20:55, 9 April 2010 (UTC)[reply]
No the confusion in that "miss" and "mister" tell us something about the person, their sex. If I was told Miss Gok Wan will see you now I would expect a female.--92.251.159.197 (talk) 23:40, 9 April 2010 (UTC)[reply]
And if, at some point in the past, I had met Shirley Povich and Michael Learned, I might be surprised to discover that the former is a man and the latter is a woman. But I would soon get over it. :) ←Baseball Bugs What's up, Doc? carrots23:54, 9 April 2010 (UTC)[reply]
Yes, but conformism is a form of ontological confusion - equating 'what I think is right' with 'what is right'. kind of a pandemic problem on wikipedia...--Ludwigs2 21:03, 9 April 2010 (UTC)[reply]
To address the original question:
...I wouldn't get away with that, so why is he allowed to call himself this?
OK, the basic problem is that the premise of the question is false. The OP could, in fact, "get away with" calling himself anything he wants to, as long as it's not being used for fraudulent purposes. Elvis Presley was called "The King of Rock and Roll". Michael Jackson was called "The King of Pop". Neither of those guys had any known royal lineage. Yet they were able to "get away with it", because it was just a nickname. Likewise with "Miss Gok Wan". He is free to call himself that, and others are free to not call him that. Does the OP have any further puzzlements, or is it clear now? ←Baseball Bugs What's up, Doc? carrots21:16, 9 April 2010 (UTC)[reply]
 In The Short Reign of Pippin IV: Pépin meets a young American who asks, "What kind of king are you?" "What do you mean?" "Well, my father is the Egg King of Petaluma, and Benny Goodman is the King of Swing...." "I am King of France." "The hell you say!" —Tamfang (talk) 16:20, 10 April 2010 (UTC) [reply]
There are probably a number of people who have "Doctor" in their name who lack the advanced degrees usually associated with that title.
It's more a matter of force of personality than any sort of conspiracy. If you introduce yourself as "Doctor So-and-so" with enough confidence that no one laughs in your face, then that's your name. (If anyone says "Doctor of what?", honestly answer that it's just an affectation, or vaguely answer "Of life" or some other such question-dodging nonsense.)
"Doctor" is probably an easy one. Introducing yourself as "Miss"(if male) or "King" will be a bit harder, but persevere. Eventually people will just shrug their shoulders and humor you. (See Emperor Norton.)
Sure, it's slightly confusing to dodge convention like that, but there's no law against being slightly confusing! APL (talk) 22:54, 9 April 2010 (UTC)[reply]
The funniest thing to me about this complaint is that Gok isn't even his birth name and while it sounds like it may be a male Chinese name to me (although I'm far from an expert), I'm going to guess the majority of British people would have no idea of this were it not for Gok Wan and if it were a female name, I'm going to take a while stab that the OP would still be complaining that Gok Wan should change his/her name. I should also point out that although some of the above statements appear to be presuming Gok Wan has a female sexual identity, this may be the case, but I haven't seen much evidence for this and a quick search isn't providing much supporting evidence. I think people are reading too much into one comment from one show which may or may not have been intended as a joke and we have no real indication that this is how he/she prefers to be identified. Perhaps he/she doesn't even care... Nil Einne (talk) 01:19, 10 April 2010 (UTC)[reply]
Hmm. On the Internet, and being upset due to considering the someone's "misuse" of gender identification. 92.251.159.197 is likely to be upset more in future. --203.22.236.14 (talk) 06:26, 10 April 2010 (UTC)[reply]

I agree with you, OP. But as you can see, homosexuals are rather untouchable nowadays. --Belchman (talk) 10:41, 10 April 2010 (UTC)[reply]

So you think it would have been different if Gok Wan were heterosexual? Nil Einne (talk) 05:51, 11 April 2010 (UTC)[reply]
Yea, Damn our liberal society for not stopping people from choosing their own name! Who decided that people should have even the smallest modicum freedom? We need to put a stop to that.
Normally, we would find a way to strip this person of the basic human right to define their own identity, but since this person is a minority we can't. Stupid political correctness!
Seriously though, if it makes you feel any better, If you ever meet this person, you're free to call him whatever you like. I suggest "Doctor". That'll really confuse him. APL (talk) 05:13, 12 April 2010 (UTC)[reply]

Did you consider that there may have been an element of piss-taking in the introduction? --TammyMoet (talk) 14:24, 10 April 2010 (UTC)[reply]

I'm surprised that no one has mentioned Dame Edna Everidge (fictitious sex and title). —Tamfang (talk) 16:20, 10 April 2010 (UTC)[reply]

Excellent point. For that matter, what about Mrs. Doubtfire? ←Baseball Bugs What's up, Doc? carrots16:26, 10 April 2010 (UTC)[reply]
Less interesting imho; that character would not expect to be addressed as "Mrs Doubtfire" (what kind of a silly name is that anyway?) by people who know his real name. —Tamfang (talk) 06:39, 11 April 2010 (UTC)[reply]
This is not uncommon. The gay community has taken back various terms in a sense of empowerment. It's frequently heard gay men referring to each other as "fags", "queens", and "princesses", "sluts". While these may have positive or negative connotations, they are not used in the homophobic sense. Effeminent men may use "Miss" and "bitch" to refer to each other for the same reason. See sexual slurs. --Kvasir (talk) 22:41, 12 April 2010 (UTC)[reply]

Interesting. I think this is a matter of definition. There are proper nouns ("names" for unique entities) and common nouns (words for general concepts). Since proper nouns have little or no conceptual content, a person is free to choose a proper noun to be designated with in his language (John, Batman, Australia or Kmdfjk), though an unconventional choice ignoring the small elements of conceptual content may lead to sneers. Since common nouns do have a lot of conceptual content, s/he is generally not free to choose what common noun people should be applied to him/her - firefighter, drunkard, elephant, pleonasm, woman. In the case of a gay or transsexual person, his/her definition of "woman" may be "someone who identifies as a lady / Miss", while others' definition may be "someone who has a vulva and no penis" (hence including transsexuals but not gays), or "someone with an XX chromosome pair" (hence including neither). Now, a person can't require others to change their definitions of words, that would be an encroachment upon their freedom to define and conceptualize their world - though they may still choose to use "elephant" or "woman" in reference to him/her as a matter of courtesy and as a kind of "idiom", even though they don't regard him/her as being technically an elephant in the literal sense. The issue is complicated by the fact that words such as "Mr/Miss" are borderline between common and proper nouns - they are used as additions to or substitutes of names, and have almost as little semantic content as proper nouns.--91.148.159.4 (talk) 15:56, 13 April 2010 (UTC)[reply]

Looking for reliable ear buds

My teenager constantly breaks the ear buds I buy for his iPod, often from pulling on the cords. A couple of times he cranked up the volume so high that the speakers went bad. I want to find a set that will last him years instead of months. Is there such a creature? Hemoroid Agastordoff (talk) 17:29, 9 April 2010 (UTC)[reply]

Listening to them at the highest possible volume is extremely bad for one's hearing, my iPod Nano is usually set at about at third of max volume.--92.251.159.197 (talk) 17:34, 9 April 2010 (UTC)[reply]
I suspect that if you make him pay for replacements, they will last much longer. StuRat (talk) 17:38, 9 April 2010 (UTC)[reply]
I'm with StuRat. If he's breaking them, make him buy the replacements. Then, he will work to make them last longer. If that means he has to wait longer because he doesn't have the money right now, then so be it. Last I checked, having a functioning personal music player was pretty high up on Maslow's Hierarchy of Needs, and as such, I don't expect his health or his psychological well being will be much affected by the lack of an ipod. --Jayron32 17:59, 9 April 2010 (UTC)[reply]

My ipod is ALAWYS at full volume and I find this is still not loud enouigh, head phones come and go the best thing to do is buy expensive ones, I have had hundreds over the yaesr but the ones I have at the moment, sennheiser are the best I have ever had, in quality of sound, durability, material, the wire cannot get tangled. brilliant, I do not want to advertise but will never use any other brand again, although I am sure they will not last for ever and I will need to get a new pair at some stage. —Preceding unsigned comment added by 62.172.58.82 (talk) 18:19, 9 April 2010 (UTC)[reply]

Health warning - that kind of behaviour is likely to make you go deaf. 78.147.131.74 (talk) 21:13, 9 April 2010 (UTC)[reply]
Absolutely. If he has it up all the way, maybe he's already had hearing loss and is compensating for it. It would be best to get to a hearing doctor and get tested. You can't fix hearing loss, but you can keep it from getting worse. ←Baseball Bugs What's up, Doc? carrots23:00, 9 April 2010 (UTC)[reply]
You could check out Bose headphones (somewhat expensive) for sound quality and durability, or you find very cheap headphones at most retail stores if you just want to replace them frequently at low cost. good luck, hope this helps...(since you didn't ask for parental advice or the effects of listening to music of high volume.) 10draftsdeep (talk) 20:14, 9 April 2010 (UTC)[reply]
I found earbuds at $1.50 a pair in WalMart - at that price, it really doesn't matter how long they last. SteveBaker (talk) 02:32, 10 April 2010 (UTC)[reply]
Don't subsidize your teen's future hearing loss. If he is putting them at a level that can blow our headphones, he is listening to them at extremely unsafe levels. Whether or not you care about that enough to encourage him not to do it, you certainly shouldn't pay for replacements. --Mr.98 (talk) 02:32, 10 April 2010 (UTC)[reply]
Possible hearing loss is an issue. I suggest you find out why he cranks the volume up so high. There is no substitute to talking with him though if he has hearing loss, you might end up having to shout :-P As for ear buds, putting the financial responsibility on his shoulders might make him take more care of his stuff. Astronaut (talk) 08:58, 10 April 2010 (UTC)[reply]
If I were the OP, I'd be quite disappointed with the responses. Apart from 10draftsdeep (and to some extent SteveBaker) nobody is attempting to answer the question, just supplying unrequested parenting advice. I too have the same problem as the OP's teenager, and I am not careless. I pay for my own in-ear headphones, and they all break within a few months. Almost all headphones, regardless of brand, have weak points where cord meets the plug or within the plug. Unfortunately, the more expensive brands may have better perceived sound quality (because for some reason audiophiles are the norm), but I am yet to find any that have build quality as a selling point. /Coffeeshivers (talk) 13:52, 10 April 2010 (UTC)[reply]
I'll answer the original question. No. If you simply want a recommendation of durable earbuds without the prescription they last years even in the face of significant mistreatment, it seems obvious from the question even without trying (and once I have tried and seeing the answers here further proving the point) that a simple internet search will provide a much more useful answer. Note that your requirements are clearly quite different from the OP, as the OP requires headphones which can survive 'cranked up the volume so high that the speakers went bad' yet 'will last him years' which I presume isn't you since you said you aren't careless. Also the wording of the OP's question suggestions we aren't simply talking about accidential tugging on the cord but careless pulling so again, not like you I presume. In addition, if the OP provides details which suggest there are much better answers then the ones you asked for, they shouldn't be surprised if people provide those answers. If they don't want such answers, they should use a commecial service which guarantees they won't provide such answers, the RD isn't it. Note that the answers aren't simply 'useless' parental advice. If he has problems with his hearing then getting him tested and the problem resolved may very well be a more effective way of getting him earbuds that will last for years. If the volume needs to be very loud because the earbuds don't fit, then getting buds which do fit similarly may very well help ensure he has earbuds which last for years. (In fact probably so would making him buy them himself or simply talking ot him, but let's not go there.) Nil Einne (talk) 07:18, 11 April 2010 (UTC)[reply]
Ear buds do not fit properly into my ear, so I find that if I am using them I must turn the Ipod up to maximum volume. They are not overly loud its just that they are meant to sit very close to your ear. Slightly more expensive headphones (rather than those useless ear buds) tend to be more ruggedly constructed. Look for thick cables (some are even made with a rope-like material) and strong looking construction of the structure. Also get him to try them out to see if he likes them.Jabberwalkee (talk) 14:08, 10 April 2010 (UTC)[reply]
I also wear out ear buds at the rate of one every couple of months. I listen to my ipod on my commute for about two hours every day and put them in and out probably at least a dozen times a day. I've started to pre-emptively wrap some tape around where the cord meets the socket, this helps them last a bit longer. I find the buds often split at the seam which I superglue back together. I've considered buying a pair of $100ish synthesizers but even if they last me 20 months instead of 2, which I highly doubt, it'll still be considerably more expensive then buying the ~$7 buds off eBay. The quality of the eBay headphones varies a LOT, I've had decent ones and really crap ones, once you find a place that has decent ones, I recommend buying up 3 or 4 at a time. Vespine (talk) 23:13, 12 April 2010 (UTC)[reply]

I got fed up with them breaking and falling out of my ears so bought hook over head phones also called ear hook headphones which have proved to be very durable and comfortable.They cost £5 in a supermarket....hotclaws 15:56, 13 April 2010 (UTC)[reply]

Stock broker jackets

Why do stock brokers wear those characteristic jackets? 71.161.45.84 (talk) 18:31, 9 April 2010 (UTC)[reply]

When there was on-floor trading (which I believe even the New York Stock exchange has finally stopped), brokers from each firm wore jackets of different colours for differentiation and identification purposes. -- Flyguy649 talk 18:35, 9 April 2010 (UTC)[reply]
And a ref. -- Flyguy649 talk 18:36, 9 April 2010 (UTC)[reply]
Do they have extra-long sleeves? Clarityfiend (talk) 22:46, 9 April 2010 (UTC)[reply]
The system that needs the coloured jackets is called Open outcry which is (or was) generally used in trading financial futures rather than stocks and shares (in London at least). Apparently the London Metal Exchange still uses it. Not my specialist subject though. Alansplodge (talk) 22:58, 9 April 2010 (UTC)[reply]
They have not stopped. There's still on floor open outcry trading for the NYSE the CBT and the AMEX, not to mention lots of other smaller boards (Philadelphia exchange maybe? not sure). PvsKllKsVp (talk) 21:03, 10 April 2010 (UTC)[reply]

u2 video, dancing with a fan

In which video can we see Bono dancing with a black fan wearing a blue top?--Quest09 (talk) 18:59, 9 April 2010 (UTC)[reply]

Possibly I Still Haven't Found What I'm Looking For. I remember some interaction with people on the street in Las Vegas in the video for that song. 10draftsdeep (talk) 20:21, 9 April 2010 (UTC)[reply]
I hoped getting a more direct answer, like a youtube link... :(--Quest09 (talk) 16:38, 11 April 2010 (UTC)[reply]
Youtube does have a nice search box. We typically don't post links here that could have copyright violation issues. 10draftsdeep (talk) 13:26, 12 April 2010 (UTC)[reply]
Yes, Youtube has a nice search box, however, if the videos are not tagged, the results are nil. Quest09 (talk) 16:31, 12 April 2010 (UTC)[reply]
It is not I Still Haven't Found What I'm Looking For Miss Bono (zootalk) 12:40, 26 June 2013 (UTC)[reply]

How many people served in both Gulf wars?

Being that the two wars were relatively close together, is it known how many people served in both? As in were on the ground in Iraq both in 1991 and 2003? 188.223.41.225 (talk) 23:14, 9 April 2010 (UTC)[reply]

Well, those wars were 12 years apart, and the number of active military personel who serve 12 years is actually quite small. Secondly, and this is becoming a problem with this current war, that number is even smaller when some of those years are spent in active combat duty; the U.S. armed forces are having a hard time finding qualified people to serve in the Captain or Sargeant ranks since those ranks generally require 4+ years of experience, and most people that end up in heavy combat don't re-up after their first contract expires. I don't know the actual number, or even how to find it, but I suspect that its not that large of a number, relative to the size of the whole military. --Jayron32 05:02, 10 April 2010 (UTC)[reply]
One who served in both wars was Saddam Hussein Abd al-Majid al-Tikriti, general and Commander in Chief of the Iraqi Mujahed Armed Forces. Cuddlyable3 (talk) 22:01, 10 April 2010 (UTC)[reply]
That's true, although saying he "served" in both wars understates matters a tad. :) I would think a number of career officers on all fronts might have been in both of them. I suspect the OP might be more interested in actual combat troops who were in both. ←Baseball Bugs What's up, Doc? carrots13:14, 11 April 2010 (UTC)[reply]
I would imagine (and I have no evidence to support this) that someone who stayed in any company for 12 years would be promoted a fair bit if you consider the number of ranks in the army, so most of the combat troops who hung around may not still be combat troops. 91.85.138.48 (talk) 19:32, 11 April 2010 (UTC)[reply]

April 10

rigoletto concertina

I picked one of these rigoletto concertina up the other day anyone know the history of this? —Preceding unsigned comment added by 24.140.117.169 (talk) 00:40, 10 April 2010 (UTC)[reply]

Have you tried looking at the WP article concertina which contains a paragraph about the history of the instrument. Richard Avery (talk) 07:24, 10 April 2010 (UTC)[reply]

Continental Divide in the Eisenhower Tunnel

Does anyone know if there's a sign marking the Continental Divide in the middle of the Eisenhower Tunnel? I've never been closer than the Loveland Pass, and I don't have all night to use Street View through the tunnel; please don't bother doing it yourself just to answer this question. Nyttend (talk) 00:45, 10 April 2010 (UTC)[reply]

This youtube video is someone driving through the tunnel (http://www.youtube.com/watch?v=BNWur19vjBk) maybe that'll be a quicker way for you to find out? ny156uk (talk) 08:17, 10 April 2010 (UTC)[reply]

I don't think so. I've driven through it before and I don't remember any sign. But I wasn't looking, and it's possible that they've added one too. PvsKllKsVp (talk) 21:01, 10 April 2010 (UTC)[reply]
This may be a tad nitpicky, but isn't the continental divide on top of the mountain? ←Baseball Bugs What's up, Doc? carrots21:54, 10 April 2010 (UTC)[reply]
Well, the location where the road passes under it could still be marked. I also don't recall seeing such a sign when I've driven through there, but that wasn't any time recently. --Anonymous, 22:13 UTC, April 10, 2010.
Thanks for the video; it's definitely faster than Street View. Yes, the Divide is on top of the mountain (picture), but I was asking about a sign underneath, as Anonymous suggested. I went to the Divide a year ago (my only time that far west :-), so now I'm quite glad that my friends agreed to go up to the Loveland Pass instead of through the tunnel :-) Nyttend (talk) 01:19, 11 April 2010 (UTC)[reply]

Creative Industry/Engineering boundary

Is there a clear boundary between creative product design and product engineering? Im assuming that the creative industry use market research etc to come up with new concepts for products while the engineering industry turns concept into reality. Thanks. Clover345 (talk) 11:40, 10 April 2010 (UTC)[reply]

The creative people can design something elegant looking that won't work at all, then the engineers design something functional, but butt-ugly. Eventually they compromise and come up with a reasonable product. StuRat (talk) 12:50, 10 April 2010 (UTC)[reply]

There is no really clear line between functions whilst a new idea is being thrashed out. Obviously each has its own speciality, but separation occurs after the proposal is operational. i.e. There is no point designing a brilliant campaign if the product specs are not known. Equally it is silly to produce a super product and then discover there is no market. So, wherever the concept originates it has to be a team effort to get it successfully on to the market. And, incidentally, there should not be a compromise. This suggests that some factors are given up in return for others. The whole new product development process should be one of co-operation.Froggie34 (talk) 13:45, 10 April 2010 (UTC)[reply]

It's totally going to depend on the industry. Take the car industry, for example - the creative folks (mostly) make non-functional (and typically, impractical) "concept cars" - which are either ignored or used only to 'inspire' the actual product. To contrast that - look at the video game business (which is what I do), and the creative and engineering people are completely working together every step of the way. SteveBaker (talk) 15:38, 10 April 2010 (UTC)[reply]

Lost contacts

Most of my friends are contacts that I know exclusively online...via Wikipedia, online games, random introductions...It has occurred to me a few times that any of them could die and I'd never find out. Or...Would it be possible for me to find out if they'd died? How much information would I need, and what would I need to do? Vimescarrot (talk) 12:17, 10 April 2010 (UTC)[reply]

If you knew their real names and their cities you could either check obituary records or contact their local coroners. Other than that, not a whole lot. Wikipedia does have a page on Wikipedia:Deceased Wikipedians, but it is certainly incomplete. --Mr.98 (talk) 14:30, 10 April 2010 (UTC)[reply]
If they are from the U.S., and you know their names and they are distinct, or your know their names and locations, you can use the social security death index.--Fuhghettaboutit (talk) 14:36, 10 April 2010 (UTC)[reply]
Thanks...How often/when does the social security death index get updated? Vimescarrot (talk) 14:49, 10 April 2010 (UTC)[reply]
And is there an equivalent for the U.K. and other countries? Vimescarrot (talk) 17:20, 10 April 2010 (UTC)[reply]
Sorry, no idea and no idea. I will say that names come up fairly quickly after deaths though, in my experience.--Fuhghettaboutit (talk) 01:23, 11 April 2010 (UTC)[reply]
Ok. Cheers. This should come in useful. *bookmarked* Vimescarrot (talk) 09:54, 11 April 2010 (UTC)[reply]

Heidi Illustrator William Sharp

I would like to find biographical information about the particular William Sharp who illustrated the 1945 Grosset & Dunlap Illustrated Junior Library edition of "Heidi" by Johanna Spyri. I have this book,(given to me on my 10th birthday in 1945). Your article about "Heidi" shows an illustration that does not match the style of the art work in my copy of the book. Apparently yours is the original book and mine is a later edition. "AskArt" online research information includes four persons by the name William Sharp, but it is unclear, and seemingly unlikely, from the art examples provided, whether any of the four is in fact the illustrator of the book I have. Sites providing information about Grosset & Dunlap, Inc. are not helpful because that company has devolved into a mass-market paperback business, apparently with no archives of the company's earlier publications. 98.248.63.139 (talk) 16:14, 10 April 2010 (UTC)[reply]

I did a few searches for him at a newspaper archives site (subscription required) and was able to confirm that he (this William Sharp) also illustrated a book called Five Little Peppers and How They Grew but nothing else. Maybe you or someone can run with that cross-referencing detail to find more.--Fuhghettaboutit (talk) 17:27, 10 April 2010 (UTC)[reply]
This site http://www.artfacts.net/en/artist/william-sharp-125751/profile.html#Biography (subscription required) seems to have a biography. All that can be read without it is that he was born in 1900 in the Austro-Hungarian Empire in Lemberg (now Ukraine), left Europe in the era of the Third Reich and died in 1961 in Forest Hills, New York City. Apart from illustrating childrens books he seems to have developed a sharp pen for the social, racial and political reality of the Cold War period. The Queens Museum in NY seems to hold a collection, so maybe you can contact them for help. --91.113.80.216 (talk) 21:07, 10 April 2010 (UTC) Oops, --Cookatoo.ergo.ZooM (talk) 21:14, 10 April 2010 (UTC)[reply]
A used-book search turns up (along with some false hits) other books that he illustrated, mainly in the 1940s, including the Heritage Press edition of Rousseau's Confessions and A Treasury of Stephen Foster. Deor (talk) 21:24, 10 April 2010 (UTC)[reply]
This site http://lambiek.net/artists/s/sharp_william.htm has a very short biography (studied art in Lemberg, worked in Berlin, left Nazi Germany in 1934, etc). --Cookatoo.ergo.ZooM (talk) 22:13, 10 April 2010 (UTC)[reply]

How to catch a wabbit

I want to know how to catch a rabbit with household items. I can't be doing stuff like buying guns, professional traps, or other things not normally found in a household. I don't want to wait in hiding for a long time or have to dig holes or anything like that. So basically I'm looking for a trap I can build without a lot of time spent out of household items. And no, to those "funny people" (wo)manning this desk, the answer "Lie in the grass and make carrot noises" is not helpful. Thanks 68.248.227.64 (talk) 17:51, 10 April 2010 (UTC)[reply]

Hey, dat always works on me, Doc! ←Baseball Bugs What's up, Doc? carrots18:15, 10 April 2010 (UTC)[reply]
Well, what "rabbit-proof" items do you have in your home that could be used to hold the rabbit ? A milk crate might work, so you could set it up with some bait under it and a stick holding it up, tied to a string. Then wait and watch and pull the string when he goes under it. You'd then need to run out and put some weight on it before he can get a paw under the edge and lift the crate enough to escape. Putting a car floor mat under it would also prevent him from digging his way out. However, what do you plan to do with the rabbit ? If you plan to take it out to the woods and release it, for example, then you need some way to seal off the bottom better, during transport. StuRat (talk) 17:59, 10 April 2010 (UTC)[reply]
My cat catches rabbits. The last one he caught he brought into the house, unscathed if a little nervous, and released him in the junk room. When I eventually found said bunny, it was a piece of cake to pick him up by the scruff of his neck and put him in a shoe box. I held it closed while I walked to the woods at the end of the school yard and released him there. However, if you'd like to eat the rabbit, may I recommend a piece of wire bent into a noose and placed at bunny head height along the rabbit run?--TammyMoet (talk) 19:06, 10 April 2010 (UTC)[reply]
Say I wanted to keep the rabbit as a pet, or eat it. —Preceding unsigned comment added by 68.248.227.64 (talk) 19:22, 10 April 2010 (UTC)[reply]
Or both Lemon martini (talk) 19:37, 10 April 2010 (UTC)[reply]
Well, if you want to keep it, you'll need a cage. Unless you already have one at home, you'd better buy one first, so your not stuck watching a rabbit under a milk crate. You could also use some types of cages as traps, like ones that have doors which slide up and down. StuRat (talk) 20:27, 10 April 2010 (UTC)[reply]
just as an aside: wild rabbits are not quite as lovable as the over-bred, super-tame decorative rabbits you get in pet stores. they are wild rodents, and can be a bit bad-tempered. --Ludwigs2 20:59, 10 April 2010 (UTC)[reply]
That's not to say that the domestic rabbit is always peaceful; I had a Black Satin buck from shortly after his weaning until his death, and he was rather unpersonable — try to coax him out of the cage (without hurting him) and he'd scratch painfully, and the only time he ever got out by himself, he returned happily as soon as he'd eaten a bit of grass less than one rabbit length away from the door of the cage. Nyttend (talk) 01:24, 11 April 2010 (UTC)[reply]

A student of Latin was flattered when his companions invited him to join their rabbit hunt. As the party lay in hiding a rabbit appeared and the student shouted Illic a lepus!. His companions chided him for frightening the rabbit away but the student protested "How could I have known the rabbit understood Latin?". Cuddlyable3 (talk) 21:32, 10 April 2010 (UTC)[reply]

Either that student was blonde or named Ole. Or both. ←Baseball Bugs What's up, Doc? carrots22:54, 10 April 2010 (UTC)[reply]
Rabbits arent rodents their lagomorphs :). But doesn't waiting to pull a string when a rabbit enters a trap count as waiting? I think I'd be there for a while before a rabbit came to sniff the bait. 68.248.227.64 (talk) 00:28, 11 April 2010 (UTC)[reply]
Soft drink can rabbit trap --203.22.236.14 (talk) 01:07, 11 April 2010 (UTC)[reply]
That link relates the story of a redneck kid who kills tame rabbits at his college and ends up with 28 bloody pelts in his dorm room. This behavior resulted in a visit from the cops, which was somehow a surprise to him, but certainly not to me. StuRat (talk) 04:14, 11 April 2010 (UTC)[reply]
Assuming you don't want a live rabbit - Snare trap says that a snare is by far the simplest - and the most effective. All you need is a length of thin, flexible wire. SteveBaker (talk) 02:24, 11 April 2010 (UTC)[reply]
Dust bunnys can of course be easier to catch. Bus stop (talk) 03:39, 11 April 2010 (UTC)[reply]
I don't know if I should tell you this as its probably a horrible way for a rabbit to die, but you could make a snare from a piece of wire, such as picture wire. The rabbit is held in terror with the snare cutting into it until you find it and kill it. Snares are also bad because they can catch other animals, perhaps foxes or cats, and if caught by a limb then they may lose the limb. 84.13.169.129 (talk) 11:01, 11 April 2010 (UTC)[reply]

questions and answers

why can't a browser give a direct, plain, English answer to a plain English question? Are we waiting for an artificial inteligence to emerge? —Preceding unsigned comment added by 89.240.89.112 (talk) 18:14, 10 April 2010 (UTC)[reply]

Given the trouble that many of us humans at ref desk have interpreting questions from users, it shouldn't surprise you that a machine would have even more trouble figuring it out. Sometimes you can go into google and type a simple question and it might take you to the right place(s), or at least get you in the ballpark so that you can do narrower searches. Do you have an example of a question you're wanting to ask the browser? Or is this just a general observation? ←Baseball Bugs What's up, Doc? carrots18:17, 10 April 2010 (UTC)[reply]
Agreed, the problem is that most people aren't at all clear when they ask a question. Consider your Q, which is better than most, but still not perfect. You didn't capitalize the first letter, which may confuse an AI looking for the start of the sentence. You used the wrong term, "browser", instead of "search engine". You also spelled "intelligence" wrong. Now a human has no problem figuring out what you meant, in this case, but an AI might. Ask Jeeves, now Ask.com, did try that approach, but they had limited success. StuRat (talk) 18:28, 10 April 2010 (UTC)[reply]
See http://www.google.com/help/features.html. -- Wavelength (talk) 18:37, 10 April 2010 (UTC)[reply]
Wolfram Alpha thinks it can... --Belchman (talk) 18:40, 10 April 2010 (UTC)[reply]
Maybe we should turn over some of the crazier questions here to that program and see what it can come up with. Even keeping it simple, I wonder what it would do with Bill Cosby's question, "Why Is There Air?" ←Baseball Bugs What's up, Doc? carrots18:46, 10 April 2010 (UTC)[reply]
Wolfram Alpha was prepared for that one, including Cosby reference! see here ---Sluzzelin talk 20:38, 10 April 2010 (UTC)[reply]
Yes and no. The original context was Cosby's girlfriend, a philosophy major, asking the question. It wasn't really about the scientific origin of molecules. It was about the great "whys" of the universe, or whatever. It was also Cobsy being silly. The next question to ask would be, "How much wood would a woodchuck chuck..." and see if they have a canned answer or if they "think about it". ←Baseball Bugs What's up, Doc? carrots22:14, 10 April 2010 (UTC)[reply]
I asked it, "Who will win the pennant in 2010?" and stumped it.[5] Back to the ol' drawing board! ←Baseball Bugs What's up, Doc? carrots22:18, 10 April 2010 (UTC)[reply]
It couldn't handle the bunny question above either. --203.22.236.14 (talk) 01:30, 11 April 2010 (UTC)[reply]
Maybe I am using it incorrectly, but it was unable to supply the answer to "Which football team won Superbowl XVIII?" It was however able to provide an answer to the woodchuck question. Googlemeister (talk) 16:29, 12 April 2010 (UTC)[reply]

Now that was entertaining...Certain search engines have predictive capabilities-if a question has already been answered,it assumes you will want to go to it again and so will make suggestions. I typed in 'Why is there air?'-on having reached 'Why is there a' it gave me a helpful list of possible alternatives.Top was 'Why is there a dead Pakistani on my couch'. Go on try it :) And no,I haven't a clue... Lemon martini (talk) 19:42, 10 April 2010 (UTC)[reply]

Artificial intelligence research used to be more oriented towards recreating human levels of intelligence (which is necessary to be able to answer questions in general). Around the time of the first AI winter, it became clear that, for the foreseeable future, no real progress can be made towards that. Currently research in those areas is focused on more limited tasks. Paul (Stansifer) 21:11, 10 April 2010 (UTC)[reply]
The BBC published a story on this on the day before this question: Why machines do not understand human speech. They don't spend much time asnwering their own question before opting instead to big up some Palo Alto Research Centre work. Still. It's almost apropos. --Tagishsimon (talk) 23:50, 10 April 2010 (UTC)[reply]
Until machines become smart enough to "read between the lines", as humans do, they will remain stupid machines. Decades ago, a teacher told us that humans were smart, slow and inaccurate; whereas computers were fast, accurate, and stupid. It remains ever thus. ←Baseball Bugs What's up, Doc? carrots01:42, 11 April 2010 (UTC)[reply]
See http://www.answers.com/. -- Wavelength (talk) 04:30, 11 April 2010 (UTC)[reply]

Why do horses get more jittery and unruly after eating oat bran than after wheat bran?

A woman who owns a stable and has been taking care of 15 horses for about two years (And thus, to me, seems fairly experienced), she told me that the horses do get noticeably more jittery and unruly after eating oat bran than after eating wheat bran.

  1. Have you seen this claim, or something similar, stated as a fact in any authoritative book about horses?
  2. Have you yourself experienced something like this, or something similar, about (your) horse(s)?
    If so, then how, by what words, would you describe the effect of oat bran on a horse?

I intend to search, in scientific research reports, to find out the reason why behind this phenomenon. Therefore I need as many keywords, facts and observations as possible.
Could you please help me?
--Seren-dipper (talk) 20:01, 10 April 2010 (UTC)[reply]

wikt:feel one's oats probably isn't scientifically helpful, but shows that it's a widely observed phenomenon. 66.127.52.47 (talk) 04:48, 11 April 2010 (UTC)[reply]

That expression was new to me, and therefore VERY helpful! Thank you!  :-)
--Seren-dipper (talk) 03:39, 12 April 2010 (UTC)[reply]
[6] uses highstrung, [7] uses hot (also [8] I think) while [9] uses heated, [10] + use fizzy; and [http://www.wowhorses.com/horse-feed-behaviour.html uses hyperactive behaviour. [11] has relevant discussion and [12] + [13] relevant research. Perhaps also [14] although only very minor. Also while not directly relevant to the question [15], [16] and [17] may be of interest. Also you may want to look into the research of Dr. Paul McGreevy. I have no particular interest in or experience with horses and found these all from the description you provided (well some from 'oats wheat horse' or 'oats wheat horse behaviour' but mostly 'oats horse behaviour' since it appears to be oats in particular that are singled out as the issue) so you probably already have enough to start your research Nil Einne (talk) 06:51, 11 April 2010 (UTC)[reply]
Yes! :-) I now have plenty! Thank you very much!
--Seren-dipper (talk) 03:39, 12 April 2010 (UTC)[reply]

One war in the whole 20th century?

I once heard about someone who claimed that instead of stuff like WWII, spanish civil war, korean war etc there was just one war 1914-89. DOes anyone know where I can read more about this? —Preceding unsigned comment added by 92.251.158.90 (talk) 20:46, 10 April 2010 (UTC)[reply]

The claim may relate to teaching of Jehovah's Witnesses that the current world era, or "system of things", entered the "last days" in 1914. Jehovah's Witnesses' doctrines on the End Times, are explained in detail in their literature. For example, they teach that the Greek word parousia, often translated as 'coming', means 'presence'. Cuddlyable3 (talk) 21:17, 10 April 2010 (UTC)[reply]
Or more in the mainstream, this[18] or this[19] "The War of the World; 1914-1989" by Niall Fergusson, Allen Lane, 2006. Alansplodge (talk) 22:15, 10 April 2010 (UTC)[reply]
Ah yes this book is what I was looking for, not that Jehovah's Witnesses thing. Thanks to both of you anyway--92.251.158.90 (talk) 22:47, 10 April 2010 (UTC)[reply]
See also the short twentieth century. — Kpalion(talk) 08:05, 11 April 2010 (UTC)[reply]

Is the airline industry considered part of public transportation?

Is the airline industry considered part of public transportation? We pay fares to ride an airplane and airports are like transit malls. WJetChao (talk) 23:50, 10 April 2010 (UTC)[reply]

Well, there is a small section on the subject within the public transportation article. The wording of it is a little odd, but it seems to suggest that what we consider "public transportation" is schedule-driven; that is, the bus stops, picks up whoever's there, and leaves. It doesn't wait long. You could think of several differences. One is that public transportation as we think of it is unreserved, "first-come, first served". Airline bookings are reserved in advance, whereas bus and commuter train are "show up and board". If that makes sense. So in that sense, AMTRAK would not be "public transportation" either. ←Baseball Bugs What's up, Doc? carrots23:58, 10 April 2010 (UTC)[reply]
FFS. The whole industry isn't, but scheduled passenger services are clearly well within the definition of Public transport. See p1 & p2 in that article. --Tagishsimon (talk) 00:05, 11 April 2010 (UTC)[reply]
Not all busses are show-up-and-board; Greyhound encourages buying well in advance, and you pay far more for a ticket if you buy for that day instead of buying a week or two ahead of time. Nyttend (talk) 01:25, 11 April 2010 (UTC)[reply]
Yes. Greyhound would not fit the standard definition. City busses would. Assuming there really is a standard definition. ←Baseball Bugs What's up, Doc? carrots01:38, 11 April 2010 (UTC)[reply]
Our Public transport article provides a reasonable definition: "a shared passenger transportation service which are available for use by the general public, as distinct from modes such as Taxicab, car pooling which are not shared by strangers without private arrangement." In what way would Greyhound not fit that bill? And then the article provides sections on Airlines, and Buses and coaches. There's no assertion that public transport = turn up & go, contrary to the repeated direction of your assertions. --Tagishsimon (talk) 02:02, 11 April 2010 (UTC)[reply]
I'm going by what the lead has to say about it, whose examples all seem to be the "turn up and go" variety. Yet later on it talks about airlines and such. So don't blame me for the article being unclear and self-contradictory. I am at this point uncertain as to just what the term "public transportation" is supposed to mean. ←Baseball Bugs What's up, Doc? carrots04:46, 11 April 2010 (UTC)[reply]
Uh are we reading the same article? (Quick check in the history suggests it hasn't changed much.) From the lead (second and third paragraph):
Public transport modes include buses, Ferrys, trams and trains and 'rapid transit' (metro/subways/undergrounds etc). Intercity public transport is dominated by airlines, coaches, and intercity rail. high-speed rail networks are being developed in a many parts of the world.
Most public transport runs to a scheduled timetable with the most frequent services running to a headway. Share taxi offers on-demand services in many parts of the world and some services will wait until the vehicle is full before it starts. Paratransit is sometimes used in areas of low-demand and for people who need a door-to-door service.[1]
How exactly does this imply that it all are the turn up and go variety? Most of the examples for intercity transport are clearly not the turn up and go variety. Even in the first sentence of that paragraph, buses, ferries and trains are not always the 'turn up and go' variety. Note also key words like 'include', 'most' etc.
The article is no FA but the lead seems decent enough other then some minor issues (like the capitalisation of ferrys and the non capitalisation of high speed which I've fixed). It first gives a resonable definition of what public transportation is that Tagihsimon has already mentioned. It then goes into more details, including giving examples.
Nil Einne (talk) 06:13, 11 April 2010 (UTC)[reply]
Common carrier is a more precisely defined term which the OP may be interested in. FiggyBee (talk) 01:37, 12 April 2010 (UTC)[reply]

April 11

Is this a boil, a wart, a bite, something else?

I know there's a restriction against medical advice, but hopefully asking what this inflammation could be doesn't really count. If anyone could clarify it for me, it'd be great. http://imgur.com/B8683.png 202.10.95.178 (talk) 03:02, 11 April 2010 (UTC)[reply]

<Removed questionable diagnosis Nil Einne (talk)> See your doctor —Preceding unsigned comment added by 68.248.227.64 (talk) 03:08, 11 April 2010 (UTC)[reply]
It could be anything. None of us here are dermatologists. See as noted above, go see a doctor. ←Baseball Bugs What's up, Doc? carrots04:48, 11 April 2010 (UTC)[reply]
Removed questionable diagnosis from 68 Nil Einne (talk) 08:09, 11 April 2010 (UTC)[reply]

Most effective bike lock

What is the strongest type of bike lock? With these sorts of things they get better if you pay more but is there a design which is the strongest? Thanks. Chevymontecarlo. 10:09, 11 April 2010 (UTC)[reply]

Good article here going through pros and cons of the different lock types.Dalliance (talk) 10:34, 11 April 2010 (UTC)[reply]
http://www.whycycle.co.uk/safety_and_security/bicycle_locks/

Great! Thanks! :) Chevymontecarlo. 10:44, 11 April 2010 (UTC)[reply]

One problem is that the wheels of a bike come off easily. So, you don't want to just chain the wheels. If possible, chain the frame to a heavy object. (You may also want to chain the wheels to that object, to prevent people from stealing them.) StuRat (talk) 13:48, 11 April 2010 (UTC)[reply]
You might recall Carl Bernstein (Dustin Hoffman) in All the President's Men, in which he had a bicycle tire next to his desk. This supposedly reflected Bernstein's real-life habit of detaching his front tire as a deterrent against theft. ←Baseball Bugs What's up, Doc? carrots14:05, 11 April 2010 (UTC)[reply]
That wouldn't prevent someone from tossing the rest of the bike into a truck and adding a wheel later. I'd also think that unscrewing the wheel several times a day would result in stripping the threads, eventually. StuRat (talk) 14:12, 11 April 2010 (UTC)[reply]
Quick release --Phil Holmes (talk) 16:59, 11 April 2010 (UTC)[reply]

So you think one really long lock or multiple locks would be more effective? Chevymontecarlo. 16:08, 11 April 2010 (UTC)[reply]

All the things I've ever seen have recommended a shackle/D-style lock as the most secure. Cable and chain locks can be removed "easily" with a bolt cutter applied to the cable/chain, whereas someone needs a metal saw and a bit of time to cut through a shackle lock. (The bike shop salesman where I purchased my bike referred to cable locks as "suburban locks" - that is, enough to keep a bored kid from riding off on it, but not enough to stop a determined thief.) The normal recommendation is to attach the bike to the rack through the frame and the back wheel (because the rear cogset and derailleur are valuable). Higher end shackle locks sometimes come with an additional cable, which is long enough to loop through the front wheel as well. I've also seen people detach the front wheel and pair it alongside the rear before locking with the shackle. Other things to possibly worry about are the handlebars and the seat, which often can be easily removed, along with bags, lights, expensive pedals, etc. - But practically, your best bet for security lies in social factors, not massive numbers of impenetrable locks. Parking your bike in a visible, well lit location, with a number of people around works better than spending $50 for an 'extra lock. (The first lock is a social factor too. Other people know that a person going at a lock with a bolt cutter or hacksaw is a thief, whereas a person just walking up and riding off could just be the owner.) Having a bike not worth stealing/you can afford to replace works well too. If you have a $2000+ race bike, you may want to rent a bike locker or bring the entire bike inside with you (if permitted). The benefit of an inexpensive bike is that you don't end up looking like Mr. Bean, taking parts of you vehicle with you, lest it get stolen. -- 174.24.193.122 (talk) 18:15, 11 April 2010 (UTC)[reply]
The advice I've heard is that the two major sorts are chain+lock and U-lock. Use whichever is least common on expensive bikes in your area. Theives can easily break a chain+lock with heavy bolt cutters; they can easily break a U-lock with an car jack. It is unlikely that any one criminal will carry both.
The other advice I've heard is use a cheap lock and ride an old bike. --Polysylabic Pseudonym (talk) 02:25, 12 April 2010 (UTC)[reply]

OP: Your first sentence asks a significantly different question than the section title, i.e. strongest vs. most effective. There's good discussion on strongest above; for the other, the most effective lock needs only to be better than the one on the bike next to yours. DaHorsesMouth (talk) 23:45, 11 April 2010 (UTC)[reply]

Oh, and per DaHorsesMouth above, one of the best things you can do with bike car our house security is ensure yours is better secured, less attractive or less valuable than the one it's (parked) next to. --Polysylabic Pseudonym (talk) 02:29, 12 April 2010 (UTC)[reply]
"I don't have to outrun the bear. I just have to outrun you." --jpgordon::==( o ) 03:56, 12 April 2010 (UTC)[reply]

Property

Are any of the following houses real? By that I mean bit just sets on television shows, can you tell me who they are owned by and whether or not it would be possible to but them, Nile's house from Fasier, or Frasiers apartment, alternativley the beech house in Two and a half men. Thank you —Preceding unsigned comment added by 82.3.145.145 (talk) 13:22, 11 April 2010 (UTC)[reply]

The way it normally works is that they use a real house for external establishing shots, while the interior is all shot on a set (some scenes "right outside the door" are also done on set). Thus, the real interior may not match the set. There have been movies shot inside real buildings, but they often need to rip out walls and such to make room for the cameras. In the movie Taxi Driver, they tore up a building for an interior shoot and it almost collapsed on them, due to loss of structural supports. StuRat (talk) 13:41, 11 April 2010 (UTC)[reply]
Typically, for TV at least, you can tell that the interior shots are done on a stage, especially when filmed in front of a live audience. The stage sets are often much "roomier" than the real places. One example is Cheers. On the night of its last first-run episode, the show's players were in the real Cheers bar's interior, and it seemed very cramped. In short, it seemed like a real bar. With movies, there is more flexibility. In Field of Dreams, the interior of the house was used, although it took some remodeling to make it work. ←Baseball Bugs What's up, Doc? carrots14:03, 11 April 2010 (UTC)[reply]
Also, when using a set you only see 3 walls, since one wall is absent for the cameras. They could have a duplicate set with the missing wall or they could alter the existing set, if necessary, but time and money constraints rarely allow for this. StuRat (talk) 14:17, 11 April 2010 (UTC)[reply]
For films that actually contain a stage and an audience, they often film it twice: once from the audience viewpoint, and again from the stage viewpoint with the audience in the background. They are of course edited together to make it appear it's all happening at the same time. Another example of indoor and outdoor sets would be Stanley Kubrick's version of The Shining. The exteriors were done at Timberline Lodge on Mt. Hood in Oregon. That was the first fakery, as it was supposedly set in Colorado. Second, they had interiors of some huge establishment. Timberline Lodge is actually small and cozy inside. ←Baseball Bugs What's up, Doc? carrots15:19, 11 April 2010 (UTC)[reply]
See this thread [20] for Frasier Crane's apartment. The view from the window implies a location on Queen Anne Hill that is apparently parkland (i.e., the place where the picture was taken to create the set's background), the theoretical address is on the other side of downtown. In other words, it is not a real place. Acroterion (talk) 19:50, 11 April 2010 (UTC)[reply]

White after Labour day

Oft times in american shows one hears the frase uttered that one should not wear white after labour day, why not? Please explain. Thank you —Preceding unsigned comment added by 82.3.145.145 (talk) 13:24, 11 April 2010 (UTC)[reply]

Fashion. StuRat (talk) 13:36, 11 April 2010 (UTC)[reply]
Or more helpfully meltBanana 16:17, 11 April 2010 (UTC)[reply]
In the film Serial Mom, Kathleen Turner's character kills Patricia Hearst's character for wearing white shoes after Labor Day. Woogee (talk) 21:45, 11 April 2010 (UTC)[reply]
That may be the best answer: you don't wear white shoes after Labor Day (note post-Webster spelling) because Serial Mom might kill you. —Tamfang (talk) 00:23, 12 April 2010 (UTC)[reply]
What I have always wondered is, when are you supposed to start wearing white again? New Year's? Spring? Adam Bishop (talk) 02:06, 12 April 2010 (UTC)[reply]
Memorial Day. --jpgordon::==( o ) 02:53, 12 April 2010 (UTC)[reply]

Japanese knife/cleaver

Anyone know what this knife is? --jpgordon::==( o ) 18:57, 11 April 2010 (UTC)[reply]

We have an article Japanese knives. Bus stop (talk) 19:06, 11 April 2010 (UTC)[reply]
Did I miss the picture of this one? I wouldn't have asked if I'd been able to find it easily. --jpgordon::==( o ) 19:14, 11 April 2010 (UTC)[reply]
No, sorry. I was just pointing it out in case anyone else wanted to be made aware of it. "Mac" Japanese knives are great. I have this "Original Series" with rounded tip and hole for hanging. Very good for preparing vegetables. Bus stop (talk) 19:17, 11 April 2010 (UTC)[reply]
I couldn't find angled knives or cleavers with similarly shaped blades on any of the Japanese kitchen knife sites I scanned. Could it be a gardening hatchet? If you do a google image search for "Japanese hatchet" you get some similar tools (some of them on survival-kit sites). ---Sluzzelin talk 05:23, 12 April 2010 (UTC)[reply]
Ah ha! Indeed, it seems to be one of these; the scabbard in this picture looks exactly like mine. The thing is wonderfully balanced; I think I may very well chop a branch or two with it. --jpgordon::==( o ) 05:42, 12 April 2010 (UTC)[reply]
I think I have seen Japanese fishmongers at fishmarkets use it to chop off the heads of larger fish.--Saddhiyama (talk) 08:42, 12 April 2010 (UTC)[reply]
The name of the knife is 鉈/なた/nata in Japanese. Oda Mari (talk) 09:06, 12 April 2010 (UTC)[reply]
Isn't that just generic for "hatchet"? --jpgordon::==( o ) 00:45, 13 April 2010 (UTC)[reply]

Hotel electronic key cards.

Is it true that these cards have embedded into their magnetic strip enough information about the client such as his/her name, address, room number, arrival and departure dates, bank account details etc., so as to allow an unscrupulous staff member to scan those details and use them for identity theft or fraudulent purposes? And if so, it would therefore be unwise to deposit the keycard at Reception upon departure? Thanks. 92.30.55.2 (talk) 19:35, 11 April 2010 (UTC)[reply]

No, they don't. -- Finlay McWalterTalk 19:41, 11 April 2010 (UTC)[reply]

"Yarmouth Roads" on board ship?

I sail occasionally on a square-rigged sailing ship. On my last trip, I did some painting work for the first time. The locker in the fo'c'sle that held the paint brushes and sundry related items was known as "Yarmouth Roads"; this was written above the door. I was told that this was the traditional name for such a locker on board ships.

I'm curious, and trying to find some reference or information about this, but a few minutes' googling hasn't turned up much. Most results are for the geographical location (which as it happens I live near, and can see no obvious link to a locker for paintbrushes), although I did find the term used to describe a large bunk in the stern of a fishing smack which provides a kind of precedent. Can anyone find anything more? 93.97.184.230 (talk) 21:53, 11 April 2010 (UTC)[reply]

This provides an explanation, albeit based on supposition: Yarmouth Roads is a mooring ground, and one will find the odds and ends of shipping moored there, especially during downturns in trade. The locker is named after the geographical feature because all odds and ends in the ship end up in it. --Tagishsimon (talk) 23:15, 11 April 2010 (UTC)[reply]
Excellent - thankyou. 93.97.184.230 (talk) 11:22, 12 April 2010 (UTC)[reply]

Prying an iPod out of its case

I bought a 1G iPod touch from someone, but it's been crammed into a 2G case. The ill fit has been getting on my nerves, and I'm determined to get it out. However, "a little elbow grease" just can't seem to cut it this time. How can I get this case off? The bottom can be taken off, but the top shows no signs of moving. I'm not averse to breaking the case, but I don't want to damage the iPod.--The Ninth Bright Shiner 23:12, 11 April 2010 (UTC)[reply]

Try a hair dryer -- it won't hurt anything, but acrylic has a high expansion coefficient. Might be the equivalent of running a jar under hot water to loosen the top. --jpgordon::==( o ) 02:52, 12 April 2010 (UTC)[reply]
Doesnt pointing a hair dryer to something damage its electronics due to the ionisation of the air? That's what I thought. --Ouro (blah blah) 06:45, 12 April 2010 (UTC)[reply]
Due to ionization ? No. The heat could be a problem, though, if the electronics get too hot, especially if the device is on at the time. I'd also remove the batteries, first, so they don't expand and leak. StuRat (talk) 14:07, 12 April 2010 (UTC)[reply]
Though, given the original question, is unlikely for two reasons, only one of which is unique to the OP's situation. --jpgordon::==( o ) 14:31, 12 April 2010 (UTC)[reply]

April 12

ayers rock

There used to be a book at the top of ayers rock how do I find it and is it possiable to veiw it???

Yes, there was such a book. Apparently it's no longer there, mainly because climbing Uluru is considered culturally insensitive and is officially discouraged, and so having any sort of incentive to climb would run counter to that.
As for the whereabouts of the book, assuming it still exists, maybe you could contact someone at the Uluṟu-Kata Tjuṯa National Park. -- 202.142.129.66 (talk) 02:39, 12 April 2010 (UTC)[reply]
There was such a book, and I've got a picture to prove it. (Well it's a picture of the page with my name in it, just after I signed it in 1987.) Mitch Ames (talk) 09:32, 12 April 2010 (UTC)[reply]

Country of van

Resolved

Hi, I'm trying to add a description to This photo. I know it's a Ford Transit ambulance, but I think the writing on the side is in Italian. Can anyone help? What country is this van from? Chevymontecarlo. 06:12, 12 April 2010 (UTC)[reply]

The writing on the side says "spitalul clinic de copii". That is Romanian for "Children's Hospital." (AFAIK). Avicennasis @ 06:14, 12 April 2010 (UTC)[reply]

Thanks a lot. I'll put Romania :) Chevymontecarlo. 06:16, 12 April 2010 (UTC)[reply]

Note that Romanian is also spoken in Moldova. — Kpalion(talk) 08:25, 12 April 2010 (UTC)[reply]
If you want to be even more specific, the city seems to be Oradea, as the particular hospital is identified as the Spitalul Clinic de Copii "Dr. G. Curteanu". Deor (talk) 12:19, 12 April 2010 (UTC)[reply]

OK, I have added the city to the description. Thanks. Chevymontecarlo. 15:38, 12 April 2010 (UTC)[reply]

Thoes Small Icons What Are In Line With The Title

like on protected, spoken and featured pages

when mediawiki:sitenotice is being used, they stay in line with the title

on my wiki, they dont stay in line (see here)

iv coppied

  • Common.js
  • Monobook.js
  • Common.css
  • Monobook.css

to my wiki but it just wont do it, i dont want to use "demospace" because i want to use my own custom icons

how do i get it to work?

-Sghfdhdfghdfgfd (talk) 10:25, 12 April 2010 (UTC)[reply]

You might try this question on the computing reference desk. Deor (talk) 01:33, 13 April 2010 (UTC)[reply]

Fun / Weird / Use and unuseful information

on 4chan they are known as info thereads, saved a lot of them, very interesting, can they be used at all?

-Sghfdhdfghdfgfd (talk) 10:30, 12 April 2010 (UTC)[reply]

"Used" as in "Used to make wikipedia articles"? Not likely. We have high standards for notability and (especially) we require that any fact that is likely to be disputed must be backed up with solid references in mainstream media (books, reputable newspapers, scientific journals, etc). The threads you refer to are highly unlikely to be any of those things. So, it's almost certain that this information is pretty much useless to Wikipedia. SteveBaker (talk) 13:57, 12 April 2010 (UTC)[reply]

Nuclear waste

Is it possible and/or feasible to send our nuclear waste into the sun? One could just launch a rocket loaded with the stuff with preset coordinates and launch, no worries. If this is possible why is it not done?

think it would be because of cost and time. how much would it take to bulid something for 1 use, the fuel to power it to the sun and how long it would take the rocket to get to the sun. it takes about 8mins for light so it would take a long time for a rocket -Sghfdhdfghdfgfd (talk) 12:35, 12 April 2010 (UTC)[reply]
How long it would take to get to the sun is irrelevant. It's not like we have to wait for the first one to hit before we launch the second. :) FiggyBee (talk) 12:45, 12 April 2010 (UTC)[reply]
See Radioactive waste#Space disposal Nanonic (talk) 12:36, 12 April 2010 (UTC)[reply]
(ec) Because;
A) The world's nuclear programs produce thousands of tons of high level waste a year. Just putting that much material into orbit with current technology (ie, no space elevators) would cost trillions of dollars. Pushing it out of orbit and towards the sun would cost vastly more.
B) The world's best expendable launch systems currently have a failure rate of about 5%. The consequences of 10 tons of high level radioactive waste exploding in the upper atmosphere are unknown, but it wouldn't be pretty...
FiggyBee (talk) 12:43, 12 April 2010 (UTC)[reply]
But those practical concerns aside, once (if) we have a reliable cheap method of getting stuff to orbit (such as a space elevator), launching nuclear waste into the sun becomes a reasonable option. — Lomn 12:57, 12 April 2010 (UTC)[reply]
...which is a fairly tautological statement. "Once it becomes practical, then it can become reasonable." But it's nowhere near practical, so it's not reasonable. --Mr.98 (talk) 13:02, 12 April 2010 (UTC)[reply]
Fair point, so I'll clarify the intent. Once orbital launches are safe and cheap, "shooting something into the sun" is an effective means of disposal. The problem isn't that nuclear waste falling into the sun is in some way unsafe. Contrast with "launching nuclear waste to Mars". — Lomn 13:07, 12 April 2010 (UTC)[reply]
Agreed. --Mr.98 (talk) 13:50, 12 April 2010 (UTC)[reply]
Disagreed. An orbital launch requires a speed of 18,000 mph (8 km/s). Escaping the Earth requires a speed of 25,000 mph (11 km/s). (If you haul something to a higher enough altitude first, as in a space elevator, of course the speed requirement is lowered but not the energy requirement.) But to launch into the Sun not only requires escaping the Earth but also requires canceling its orbital velocity, which is 66,600 mph (30 km/s) -- a much bigger deal. Some sort of gravitational slingshot trajectory might be able to reduce the energy requirements but would require active control for course corrections. In any case, today's waste may be tomorrow's fuel, which argues against such irrevocable disposal. --Anonymous, 19:11 UTC, April 12, 2010.
[21] says that the cost to launch 10,000kg into low earth orbit is well over $100,000,000. That's only to low earth orbit - to get out to geostationary orbit is about twice that - and prices for an orbit to get you to the sun could only be higher than that. So let's go with $100,000 per kilo of waste...versus digging a hole in Yucca mountain and dumping it there. But much worse than that is that both commercial and NASA launches have a failure rate of around 2% to 5%. So when your rocket (containing 10 tons of radioactive waste and a few hundred tons of highly explosive rocket fuel) fails...what are the consequences? Recall the shuttle disaster when the thing exploded on the way up to orbit...that was a pretty big bang. My guess is that the consequences would be about 1,000 times worse than a 10kg "dirty bomb"...which is to say "utterly devastating". Since we'd need to make well over a hundred launches a year to do what you suggest with current rocket technology, we'd have between two and five major nuclear accidents every year! It would be vastly cheaper about as harmful to simply dump the raw waste in a big pile out in the middle of New Mexico someplace! This is SO impractical that it's hardly worth discussing. SteveBaker (talk) 13:53, 12 April 2010 (UTC)[reply]
Before you go sending nuclear stuff toward the sun, watch Superman IV. :) ←Baseball Bugs What's up, Doc? carrots19:11, 12 April 2010 (UTC)[reply]

How about disposal on the ocean floor ?

If we were to dump nuclear waste in the Mariana Trench, in containers designed to dissolve quickly, would it be sufficiently diluted by the ocean to become "safe". I realize that this would be a political and PR nightmare, in any case, but I'm just asking about the scientific merit. StuRat (talk) 14:02, 12 April 2010 (UTC)[reply]

Have you considered breeder reactors?124.171.232.30 (talk) 15:31, 12 April 2010 (UTC)[reply]
Well, if your material was denser the water, I don't see how it would circulate to any great extant. Googlemeister (talk) 16:06, 12 April 2010 (UTC)[reply]
You mean like NaCl does not circulate to any great extend? Of course the idea has a certain movie potential "Atomollusc - Death from the Deep Deep!" ;-) --Stephan Schulz (talk) 16:33, 12 April 2010 (UTC)[reply]
Japan had a plan to do exactly that, except they were going to use long-lasting containers. See Challenger_Deep#Possible_nuclear_waste_disposal_site. --Sean 17:48, 12 April 2010 (UTC)[reply]
I believe the Godzilla movies used nuclear weapons testing as the source of Godzilla, so they could have even more plot fun with this. StuRat (talk) 17:54, 12 April 2010 (UTC)[reply]

So, would the radioactive elements be at detectable levels if diluted in all the world's oceans ? Would they be dangerous ? StuRat (talk) 17:54, 12 April 2010 (UTC)[reply]

You can put it in the ocean, but it is not without its risks. On the whole the idea is that dilution and isolation would be sufficient to keep it from appreciably affecting humans, which is probably true if you only consider water transport, but there are concerns that localized contamination could work its way up the food chain. There was already a lot of dumping of low-level nuclear waste in the 1940s through the 1970s, but it has basically stopped (apparently) in the 1980s, and is banned by of the Convention on the Prevention of Marine Pollution by Dumping of Wastes and Other Matter, of which the US is a party. The major argument against it is that if something did go wrong with the waste (if it turned out it was making its way up the food chain, etc.), you can't get it out again. This lack of ability for retrieval is one of the main arguments against it as a long-term strategy. --Mr.98 (talk) 23:15, 12 April 2010 (UTC)[reply]

Sailing down the Mississippi

Ok, I and my missus are retired and we have loads of time on our hands and are not terribly short of funds, though we are not terribly rich either. We have travelled extensively across many continents and cultures, by land, sea and air. And we are looking for something different for our anniversary next year, after my wife has undergone some surgery to both feet (she is a chronic arthritic and can't walk far). We have done conventional cruising and don't particularly like it (lots of stuck-up would-be-celebrity-types). And I have just finished reading James Michener's Centennial for the umpteenth time, and I have this obsessive notion to sail up or down the Mississippi. But all I can find on the Web are local 3 hour dinner cruises or one day lecture cruises out of St. Louis or similar. Is there any way I could arrange a whole river trip? Any advice welcome. Thanks. ps. We are in the UK.92.30.49.161 (talk) 12:46, 12 April 2010 (UTC)[reply]

I'm not advertising anything, and this isn't completely on the Mississippi, but here is a 14-day river/canal cruise from Chicago to New Orleans. The current economic situation seems to have hit this sort of travel hard; at least two companies have suspended operations on the Mississippi. There really doesn't appear to be much available at this time. Deor (talk) 13:37, 12 April 2010 (UTC)[reply]
Note that the coasts of the Mississippi are heavily industrialized, so you may see lots of barges and factories. I'm sure there are pleasant areas, too, but it's not all pretty. StuRat (talk) 13:53, 12 April 2010 (UTC)[reply]
It's not that bad; most of the industrialization is in the neighborhood of major cities. Deor (talk) 14:16, 12 April 2010 (UTC)[reply]
Then there are the old, abandoned, rusting truss bridges, which are downright hideous. StuRat (talk) 14:59, 12 April 2010 (UTC)[reply]
Depending on how much work you want to do, I believe that you can rent houseboats and cruise the Mississippi yourself. Buddy431 (talk) 14:21, 12 April 2010 (UTC)[reply]
Here's one such place that claims not to have any range limits. There are probably others. Buddy431 (talk) 14:22, 12 April 2010 (UTC)[reply]
Navigating the Mississippi—what with the commercial traffic (all a lot bigger than you are), negotiating locks, and such—does require a certain amount of knowledge and experience, and a person on a vacation of limited length may not want to be spending much of the time engaged in piloting a boat. Plus, the place you linked may not have a range limit, but it does appear to have a maximum time limit of seven days for returning the boat to the starting point, which would restrict you to, oh, less than a hundred miles upriver and downriver from its base in La Crosse, Wisconsin. I can't imagine that there are any companies that would allow you to rent a boat for an extended one-way trip, but I could be wrong. Deor (talk) 14:54, 12 April 2010 (UTC)[reply]
I interpret the Q as asking about cruises, not boat rentals. StuRat (talk) 14:58, 12 April 2010 (UTC)[reply]
If, given the lack of cruise options and loads of time on their hands, the posters wanted a real adventure, they might consider purchasing a powerboat in Minnesota and using it to travel down the river to New Orleans, where they could resell the boat. (Given the depressed economy in New Orleans, though, they might have better luck reselling the boat up the coast in Mobile, Alabama.) However, navigation of the Mississippi is known to be challenging. There are currents, shifting shoals, and considerable commercial traffic, as well as locks. I probably wouldn't consider it without some past piloting experience on a large river. Marco polo (talk) 18:25, 12 April 2010 (UTC)[reply]

Bible

Is there any christian religion that follows the rules set out by god in the old testament, where it states that one should not cut the hairs on the side of the head. Please note, christian, not jewis, and if not why not?

I wouldn't be surprised to find that there's some small sect somewhere trying to (pardon the pun) religiously adhere to Old Testament law, but no major flavors of Christianity do. As for why, the New Testament repeatedly de-emphasizes slavish adherence to the list of laws. I expect there are graduate-level seminary courses in this question, so I prefer not to add my own personal rationales here. You might consider going to a local church or two and asking the pastor, though -- I'd bet it's not too hard to find a few who'd have the conversation. I will note, though, that I often wonder: "do I skip the rule for <x> because it's not necessary or because it's not convenient?" It's an interesting issue. — Lomn 13:04, 12 April 2010 (UTC)[reply]
The United Pentecostal Church International does, but based on passages in Corinthians, not because of the Old Testament (the laws of which no longer needs to be followed because they were fulfilled by Christ). Rastafarians, if you count them as Christians, follow the Old Testament rules about cutting hair. Adam Bishop (talk) 13:11, 12 April 2010 (UTC)[reply]
Probably some of the Jews for Jesus crowd would satisfy those conditions. APL (talk) 17:55, 12 April 2010 (UTC)[reply]

I suppose the OP cites Leviticus 19:27 Ye shall not round the corners of your heads, neither shalt thou mar the corners of thy beard. Ye hath Moses' word that the Lord spake that unto him. Cuddlyable3 (talk) 22:35, 12 April 2010 (UTC)[reply]

Regarding the "why not" part of the question: See Acts 10 for a vision of Peter's, which is used to justify both eating formerly unclean foods and the admission into the church of formerly unclean people (i.e., non-Jews). The presence of non-Jews in the community at all was a big deal, law-wise. Also, some of the Pauline epistles cover the superiority of faith to by-the-letter law, and his discussion of circumcision might be relevant, too. (See Romans 4, for example.) Paul (Stansifer) 03:30, 13 April 2010 (UTC)[reply]

GM Crops

Is the following assumption right or wrong, please feel free to pick holes in my theory. If rich companies creat GM seeds and sell them to farmers to grow GM crops, okay. The insects that feed on them will by evolution need to get stronger to feed on them. Then poor farmers that do not have the money to buy the GM seeds will have inferior crops with super insects. This will thus increase the starvation problem in the 3rd world. Thanks

Sure, it's a plausible scenario, but I should note that you can strip out "GM" from that whole paragraph and it'll still be just as valid. Wired ran an interesting piece in February on a new strain of wheat rust that's skipped past the developed "immunity" in the world's major wheat strains. The point here, though, is that those wheat strains were hybridized traditionally some 40 years ago -- no direct genetic modification occurred. — Lomn 13:39, 12 April 2010 (UTC)[reply]
(ec) Well, it's a bit like saying that modern housing practices have (or will) resulted in a bunch of super termites to eat them, which will then wipe out the old-fashioned kind of houses, increasing the rate of house-lessness in the third world. Matt Deres (talk) 13:42, 12 April 2010 (UTC)[reply]
It depends:
A) If the GM crops are insect resistant by producing some chemical which the other crops also produce, but in lower quantities, then you are right.
B) If the GM crops are insect resistant by producing some chemical which the other crops lack, then the insects would just develop a resistance to that chemical. Therefore, this would have no effect on their ability to eat other crops, only on GM crops. StuRat (talk) 13:47, 12 April 2010 (UTC)[reply]
Your basic contention, that if farmers adopt a monoculture they will open themselves up to major dependence on seed suppliers (because even GM monocultures will probably eventually run into problems), is probably correct and is a common argument against GM crops. That's not necessarily an argument against GM crops altogether, but it is an argument against monocultures, which are often associated with GM crops. --Mr.98 (talk) 13:54, 12 April 2010 (UTC)[reply]
An insect species cannot evolve new characteristics instantly. If and when the new "super insects" eventually appear they won't necessarily want to eat the pre-GM crops their ancestors ate. Cuddlyable3 (talk) 22:22, 12 April 2010 (UTC)[reply]
It's not so much an issue of "super insects", but rather of inadvertent artificial selection by killing the insect larvae that have no resistance, while the larvae with resistance likely will survive to reproduce. With corn (maize), for example, where you plant GM crops, you also plant a portion of conventional corn, to provide what's called a "refuge" for bugs that have little or no resistance to the insecticide toxin being produced by the GM plant. So if your crop gets infested, you'll still have a decent yield, but you'll also ensure some genetic diversity in the insect population. That's the theory, anyway. ←Baseball Bugs What's up, Doc? carrots23:06, 12 April 2010 (UTC)[reply]

What is this car?

Hi, I am trying to add a description to this picture. Can anyone help me identify the car in the picture? It looks like a Toyota or Mazda, but I'm not sure. Chevymontecarlo. 15:36, 12 April 2010 (UTC)[reply]

Well, it says Mazda on the hood, if that helps...! --Mr.98 (talk) 15:42, 12 April 2010 (UTC)[reply]
[after edit conflict] Looks like a series 4 Mazda RX7 to me, compare Google Image search. -- Ferkelparade π 15:44, 12 April 2010 (UTC)[reply]
2nd gen Mazda RX-7, Series 5. That's actually the pic from the article: Mazda_RX-7#Second_generation_.28FC.29. StuRat (talk) 15:45, 12 April 2010 (UTC)[reply]

OK I'll add that to the description. Thanks. Chevymontecarlo. 15:55, 12 April 2010 (UTC)[reply]

A Capella Group?

So, I have this song on my iPod of an A Capella group doing a "remix" of Deck the Halls. I'm almost positive it is a professional group, as it sounds studio edited, professionally done, and I swear I just remember knowing they were a professional group. I can't find them on the internet though, as all my searches on every plane bring up endless results of amateurs and covers. So, I'm hoping someone here could recognize, or maybe point me in the right direction. The song is unique, as it actually portrays a recording session where two British men are the studio workers, trying to get the group to do different sounds for the song. They go from the 40s to present, then go back to the "gregorian chant" version at the end, and the British guys get continually more annoying. Any help would be MASSIVELY appreciated, as I want to revisit my old music days and find more of their stuff! Side note: if you here a song that sounds like this, stress on the British guy's phrase "Shall we?" is emphasized more and more throughout the song; its a dead give away for the group I'm looking for. Thanks in advance! Hubydane (talk) 18:49, 12 April 2010 (UTC)[reply]

Sounds fantastic. I look forward to viewing it when someone tracks it down. Edison (talk) 01:48, 13 April 2010 (UTC)[reply]

April 13

Hemp powder

how safe is hemp protein powder? any side effects? Will taking affect drug testing?

This sounds like medical advice, which we are not allowed to give. It's possible the hemp article will have some info on that, though. ←Baseball Bugs What's up, Doc? carrots02:26, 13 April 2010 (UTC)[reply]
Psud whinging about ←Bugs jumping to conclusions about medical advice
It's no more medical than "how safe is soap? Will washing myself with soap have any bad effects?". They're not asking for a diagnosis, prognosis or even about an illness or disease. --Polysylabic Pseudonym (talk) 03:44, 13 April 2010 (UTC)[reply]
And further to that, see Kainaw's criterion: "Can the question be answered completely without providing a diagnosis, prognosis, or treatment advice?"
Diagnosis: None requested.
Prognosis: (likely outcome of an illness) Nope. No illness mentioned.
Treatment: Nope.
Bugs, I suggest you add the Kainaw's criterion link above to your favourites and refer to it before labelling a question as medical. sorry bugs, bee in my bonnet regarding people marking questions as medical, legal etc. --Polysylabic Pseudonym (talk) 04:18, 13 April 2010 (UTC)[reply]
To the OP: According to this newspaper article it's very unlikely that you could consume enough hemp protein powder to trigger a positve drug test result. See the article for details. I can't find anything claiming that it is dangerous (in comparison to other protein supplements) --Polysylabic Pseudonym (talk) 04:18, 13 April 2010 (UTC)[reply]
This FAQ should be helpful (though of course it's from a manufacturer of hemp protein powder.) --jpgordon::==( o ) 04:19, 13 April 2010 (UTC)[reply]